You are on page 1of 66

CUPRINS

Din partea redaciei


__________________________________________
Andrei Ungureanu concursuri 2003
________________________
American Mathematic Contest
______________________________

1
2
4

Note matematice
Problema celor dousprezece monede
________________________
Funciile spline ca abordare a analizei numerice elementare ______
O metod de rezolvare a ecuaiei de gradul al treilea
____________

5
10
13

Teme pentru grupele de performan


Clasa a V-a
Numere raionale pozitive
____________________________________
Clasa a VI-a
Ptrate perfecte ________________________________________________
Clasa a VII-a
Ecuaii diofantice ________________________________________________
Teorema lui Pitagora generalizat
______________________________
Clasa a VIII-a
Partea ntreag i partea fracionar a unui numr real
______
Probleme de ordonare __________________________________________
Clasa a IX-a
Funcia parte ntreag, funcia parte fracionar __________________
Clasa a X-a
Ecuaii exponeniale i logaritmice ______________________________
Clasa a XI-a
Asupra determinanilor matricelor formate numai cu -1 i 1 ______
Clasa a XII-a
Asupra unor funcii cu proprietatea lui Darboux __________________
Despre polinoamele ciclotomice
______________________________

-0-

15
17
20
23
26
29
32
38
41
43
47

Probleme propuse
Teste bacalaureat
Clasa a V-a
Clasa a VI-a
Clasa a VII-a
Clasa a VIII-a
Clasa a IX-a
Clasa a X-a
Clasa a XI-a
Clasa a XII-a

____________________________________
____________________________________
____________________________________
____________________________________
____________________________________
____________________________________
____________________________________
____________________________________
____________________________________

51
56
56
57
58
59
60
61
62

Rubrica rezolvitorilor

____________________________________

64

EDITORIAL
Din partea colectivului redacional

SSM
H

Stimai colegi i dragi elevi, iat c ncercarea noastr de a provoca


micarea matematic mehedinean la elaborarea propriei reviste intr ncet
ncet n normalitate, i prin grija Filialei Mehedini a Societii de tiine
Matematice din Romnia i prin colaborarea dumneavoastr putem astzi
s salutm apariia numrului trei al revistei. Ne bucur faptul c printre
colaboratori apar nume noi, tineri colegi de-ai notri care trimit materiale
spre publicare i tot mai muli elevi citesc revista noastr trimind
probleme rezolvate. Acestora din urm le recomandm s urmreasc cu
atenie i rubrica Teme pentru grupele de performan din care i vor
putea imbogi bagajul de cunotine matematice. Rugm de asemenea pe
colegii notri s urmreasc aceste teme din numerele anterioare pentru a
evita temele deja abordate. Domnii profesori ai cror elevi i manifest
dorina de a rezolva probleme din revist ne vor trimite listele cu numele
acestor elevi dup ce n prealabil au verificat corectitudinea rezolvrilor.
Reamintim c revista i propune printre altele popularizarea activitilor
legate de matematic ale elevilor i cadrelor didactice mehedinene; deci,
stimai colegi, v asteptm cu informaii n legtur cu participarea la
concursuri de matematic, instruiri, schimburi de experien .
De la apariia ultimului nostru numr din iunie 2003 s-au desfurat
o serie de aciuni, multe din ele putnd fi cunoscute din paginile acestei
reviste : Tabra internaional de la Poiana Pinului (Cangurul), Olimpiada
Internaional pluridisciplinar desfurat n acest an n Federaia Rus,
Tabra Internaional Cangurul din Lituania, Concursul Rezolvitorilor
Gazetei Matematice de la Piteti, Concursul interdisciplinar Vrnceanu Procopiu matematic-fizic, de la Bacu, Concursul Interjudeean Gheorghe
Pun de la Rmnicu-Vlcea, Cercul Pedagogic al profesorilor de matematic
de la Liceul tefan Odobleja, American Mathematical Competition (AMC 8)
cu participarea a 40 de elevi din CNT de la clasele VII i VIII, Conferina
Naional a Societii de tiine Matematice din Romnia la care au
participat ca delegai domnii profesori: Patrcoiu Constantin, Ciniceanu
Gheorghe, Ungureanu Mihai Octavian, Sceanu Victor, domnii Ptrcoiu i
Ciniceanu fiind alei membrii ai Consiliului Naional al SSMR. Menionm
c judeul Mehedini este considerat ca filial foarte activ a SSMR avnd
peste 100 de membrii cotizani i peste 450 de Gazete Matematice
distribuite n coli.
Din pcate, n aceast perioad au trecut n nefiin doi dintre cei
mai buni colegi ai notri, crora le vom pstra o venic amintire:
PROF. DORU P. FIRU i PROF. TEFAN BOJINC.
Dumnezeu si odihneasc.

-1-

EDITORIAL
Rezultate remarcabile ale elevului Andrei Ungureanu
la olimpiadele si concursurile internationale - 2003
Prof.Manuela Prajea
i n vara anului 2003 olimpicul Andrei Bogdan Ungureanu (clasa a IX-a
Colegiul Naional Traian) a reprezentat cu succes Romnia la Olimpiada
Internaional TUYMAADA din Iakutsck Rusia, unde a obinut Premiul III
Medalia de Bronz i la Concursul Internaional de Matematic Kangourou
Lituania, unde a obinut Premiul I.
Selecia lotului reprezentativ al Romniei pentru participarea la olimpiadele
internaionale desfurate n Japonia, Rusia i Turcia s-a realizat n urma celor
cinci probe de baraj susinute n perioada martie-mai 2003 astfel: primii ase
clasai au participat la Olimpiada Internaional din Japonia i la Balcaniada din
Turcia (seniori), iar urmtorii trei clasai, avnd limita superioar de vrst 18 ani,
au participat la Olimpiada Internaional Pluridisciplinar din Rusia (probele de
baraj sunt constituite din subiecte comune pentru elevii claselor IX-XII care au fost
selectai dup desfurarea Olimpiadei Naionale de Matematic - dup primul
baraj, Andrei s-a clasat pe locul doi). n Iakutsck, Romnia a participat cu un lot
compus din 11 elevi: matematic (Chiril Cezar, Musta Ioana, Ungureanu Andrei
medalii de bronz), fizic 3 elevi, informatic 2 elevi, chimie 3 elevi. Echipa
Romniei a ajuns n Iacutsck dupa 12 ore de cltorie cu avionul (BucuretiBudapesta Moscova-Iakutsck) i nu a fost simpl nici adaptarea la fusul orar i la
condiiile climaterice (vecintatea cercului polar), sau la faptul c soarele apunea la
orele 24:00 i rsrea n jurul orelor 3:00, astfel nct, dup o noapte de odihn
elevii romni s-au trezit n jurul orelor 4:00, emoionai i n form pentru
urmtoarele dou zile de concurs.
Concursul a constat n susinerea a dou probe a cte 5 ore i, n pofida
concurenei unor ri cu mare tradiie n matematic (China, Bulgaria, Coreea,
Germania, Marea Britanie, etc - n total 15 ri participante) elevii notri au fost
ntrecui de colegii lor bulgari, care au acaparat medaliile de aur i argint.
n afar de ineditul geografic al acestei provincii a Rusiei, participanii au
beneficiat i de o croazier pe Lena pe parcursul a dou zile cu nnoptare pe vapor,
de excursii i vizite ale obiectivelor turistice specifice acestei zone ndeprtate a
mapamondului, experiene de neuitat de care i vor aminti cu nostalgie tot restul
vieii, dup cum ne mrturisete Andrei.
n ceea ce privete participarea la Concursul Internaional Kangourou din
Lituania, lotul Romniei a fost constituit din cei ase elevi clasai pe locul nti pe
fiecare nivel (VII-XII) la barajul din aprilie 2003 Bucureti (patru din cei ase elevi
fac parte din lotul internaional de matematic). Din rile participante mentionm
Belarus, Ucraina, Austria, Germania, Rusia, Estonia, Lituania, Letonia,
etc.Concursul s-a desfurat pe parcursul a patru ore, timp n care elevii trebuiau
s rezolve ase probleme (nivel de dificultate asemntor cu olimpiadele
internaionale; domeniul preferat : combinatorica). Andrei a obtinut Premiul I, iar
Premiul II a fost obinut de Kreindler Gabriel (Romnia). Sediul Kangourou din
Lituania a fost organizat ntr-un campus la 50km. de Vilnius, unde s-au desfurat
activiti sportive: baschet, volei, ah, etc., trasee turistice, excursii n capital i
imprejurimi, s-a vizitat turnul TV din Vilnius, observatorul astronomic.

-2-

EDITORIAL
Premii obinute de elevii mehedineni la concursurile
internaionale i naionale de matematic,
iunie decembrie 2003

SSM
H

Un bilant al concursurilor matematice nationale si internationale la


care au participat elevii nostri s-ar sintetiza cronologic astfel :
1. Olimpiada Internationala de Matematica TUYMAADA- Rusia
Ungureanu Andrei Bogdan - Premiul III - Medalie de Bronz
clasa a IX-a, Colegiul National Traian (prof. Manuela Prajea)

2. Concursul International Kangourou Lituania

Ungureanu Andrei Bogdan - Premiul Iclasa a IX-a, Colegiul National Traian (prof. Manuela Prajea)

3. Tabara cu participare internationala KangourouPoiana Pinului


Buzau,
-

Concursul M.C.M. : Premiul III Echipajul jud. Mehedinti + Bucuresti :


Bazavan Eduard cls.a IX-a, Liceul Gh. Titeica (prof. Daniel Stretcu)
Janca Andrei cls.a IX-a, Liceul Gh. Titeica (prof. Daniel Stretcu)
Peptan Alexandru - cls.a XI-a, C.N.Traian (prof.Dana Paponiu)

4. Concursul Rezolvitorilor Gazetei Matematice Pitesti


-

Raveanu Alexandra Mentiunecls a VII-a C.N.T. (prof.Gh.Cainiceanu)


Rosu Stefan Mentiune cls.a VIII-a C.N.T. (prof.Gh.Cainiceanu)
Tantaru Mihai Mentiune cls a X-a C.N.T. (prof.Gh.Cainiceanu)
Pit-Rada Cosmin Premiul I cls a XI-a C.N.T. (prof. Dana Paponiu)

5. Concursul Recreatii Matematice Iasi

- Bazavan Eduard Mentiune cls.a IX-a, Liceul Gh. Titeica


(prof. Daniel Stretcu)

6. Concursul Gh.Vranceanu - Procopiu - Bacau


-

Bocse Bogdan Mentiune (Concursul propriu-zis)


clasa a IX-a, Colegiul National Traian (prof. Dana Paponiu)
Ungureanu Andrei Bogdan- Premiul I (Concursul propriu-zis)
clasa a X-a, Colegiul National Traian (prof. Manuela Prajea)
Ungureanu Andrei Bogdan- Premiul I (Baraj tip olimpiada internationala)
clasa a X-a, Colegiul National Traian (prof. Manuela Prajea)

7. Concursul Nicolae Paun Valcea


-

Nicolescu Alexandra Mentiune cls a VII-a Colegiul National Traian


(prof. Manuela Prajea)
Bobiti Ruxandra Premiul II cls a IX-a Liceul Gh. Titeica
(prof. Irina Zaman)
Ungureanu Andrei Bogdan Premiul I cls aX-a Colegiul National Traian
(prof. Manuela Prajea)
Bazavan Eduard Mentiune cls a X-a Liceul Gh. Titeica
(prof. Daniel Stretcu)

8. Concursul Gh. Dumitrescu Craiova


-

Bazavan Eduard Mentiune cls a IX-a Liceul Gh. Titeica


(prof. Daniel Stretcu)

9. Concursul Scoala cu ceas Craiova


-

Crutan Bogdan Premiul III cls a VII-a Sc. nr. 6 (prof. Anghel Badaluta)

Colectivul de redacie

-3-

EDITORIAL
AMERICAN MATHEMATICAL COMPETITION
AMC 8 COLEGIUL NAIONAL TRAIAN LA A DOUA
PARTICIPARE
Prof.Gh. Ciniceanu
i n aceast toamn, pe 19.XI.2003 elevii C.N.T. au participat la
Olimpiada de matematic a SUA adresat elevilor de gimnaziu clasele a VII-a
i a VIII-a. S-au nscris 40 de elevi, 21 de la clasele a VII-a i 19 de la
clasele a VIII-a. Elevii au avut de rezolvat 25 de probleme n 40 de minute,
aceleai pe care n aceeai zi elevii americani i din alte coluri ale lumii leau primit. Problemele sunt primite n colet sigilat de la sediul AMC din
Nebrasca SUA, toate formulate n limba englez i cu 5 variante de rspuns.
Iat un exemplu din concursul pe anul 2002 pentru care avem aprobarea de
copiere :
Harold tosses a nickel four times.The probability that he gets at least as
many heads as tails is
A) 5/16
B) 3/8
C) 1/2
D) 5/8
E) 11/16.
Loki ,Moe,Nick and Ott are good friends. Ott had no maney , but the
others did. Moe gave Ott one-fifth of his money, Loki gave Ott one-third of his
money. Each gave Ott the same amount of money. What fractional part of the
groups money does Ott now have?
A) 1/10
B)1/4
C) 1/3
D) 2/5
E) 1/2.
Rezultatele obinute de elevi au fost urmatoarele (din 25p)
Tuta Leontin
VIII Pr. I
21p
Raveanu Ioana
VIII Pr. I
20p
Slama Victor
VIII Pr. I
20p
Dutulescu Leonard
VIII Pr. II
19p
Coanda Oana
VIII Pr. II
18p
Rosu Maria
VII
Pr. II
18p
Samciuc Andrei
VII
Pr. II
18p
Cainiceanu Andrei
VIII Pr. III
17p
Calianu Alina
VIII Pr. III
17p
Negrea Nicolae
VIII Pr. III
17p
Trocan Irina
VII
Pr. III
17p
Media realizata la clasa a VIII-a este 15,8 iar la clasa a VII-a 11,8.
Consideram ca acest concurs a fost o reusita , elevii participand cu
placere si dorinta de autoverificare atat la matematica dar si la limba
engleza.
Colegiul nostru a fost invitat sa participe si la AMC 10/12 din 10
feb.2004, fiind inscrisi in concurs 100 de elevi.

-4-

NOTE MATEMATICE
Problema celor dousprezece monezi

SSM
H

prof. Pi-Rada Marica


prof. Pi-Rada Ionel-Vasile

Enun: Se dau dousprezece monezi, unsprezece monezi identice


(avnd aceeai greutate standard) i nc o moned diferit de celelalte doar
prin greutate. Utiliznd o balan negradat i fr greuti marcate s se
determine din trei cntriri moneda diferit i s se precizeze dac este mai
uoar sau mai grea fa de greutatea standard.
Pentru rezolvarea acestei probleme vom generaliza enunul n mai
multe variante. Pe parcursul rezolvrii vom lucra cu sisteme formate din
patru mulimi (A, B, C, D) avnd urmtoarea semnificaie:
- A={a1, a2, } este mulimea monezilor date, despre care nu se cunoate
nici o informaie;
- B={b1, b2, } este mulimea monezilor despre care se tie c nu pot fi
mai grele dect greutatea standard;
- C={c1, c2, } este mulimea monezilor despre care se tie c nu pot fi
mai uoare dect greutatea standard;
- D={d1, d2, } este mulimea monezilor despre care se tie c au
greutatea standard;
Este uor de observat c cele patru mulimi sunt disjuncte.
Voi formula n continuare cinci probleme cu monezi, n fiecare din ele se
cere determinarea monezii cu greutatea nestandard precum i determinarea
faptului dac ea este mai grea sau mai uoar fa de greutatea standard.
Fiecare din problemele urmtoare se vor putea rezolva utiliznd n, n1,
cntriri cu o balan negradat i fr uniti de msur marcate.
Pentru a enuna cele cinci probleme este suficient acum s precizm doar
sistemele de patru numere naturale (|A|, |B|, |C|, |D|), reprezentnd
numrul de elemente corespunztoare mulimilor A, B, C i D.
Iat enunurile celor cinci probleme:
P(1, n)=((3n-3)/2, 0, 0, 0),
P(2, n)=((3n-1)/2, 0, 0, 1),
P(3, n)=( 0, (3n-1)/2, (3n+1)/2, 0),
P(4, n)=( 0, (3n+1)/2, (3n-1)/2, 0),
P(5, n)=( 0, (3n-1)/2, (3n-1)/2, 1).
Prin "1" , respectiv "0" dorim s semnificm faptul c este vorba de un
numr 1, respectiv 0. n problemele P(1,n) i P(5,n) este esenial n
rezolvare ipoteza "1".
Vom trata n continuare fiecare din cele cinci probleme artnd c utiliznd
o singur cntrire problemele i vor reduce dimensiunea (din n n n-1).
n cele ce urmeaz rezultatul unei cntriri l notm prin s<d , s=d respectiv
s>d , unde s reprezint greutatea din talerul stng, iar d reprezint
greutatea din talerul drept.
Problema P(1, n):
Notm p=(3n-1-1)/2 i observm c |A|=3p.
Aezm pe talerul stng monezile a1, a2, , ap, pe talerul drept monezile
ap+1, ap+2, , a2p i rmn neaezate pe talere monezile a2p+1, a2p+2, , a3p.
Dac s<d, atunci avem de rezolvat problema P(5, n-1).

-5-

NOTE MATEMATICE
Dac s=d, atunci avem de rezolvat problema P(2, n-1).
Dac s>d, atunci avem de rezolvat problema P(5, n-1).
Problema P(2, n):
Notm p=(3n-1-1)/2 i observm c |A|=3p+1.
Aezm pe talerul stng monezile a1, a2, , ap i o moned bun d1, pe
talerul drept monezile ap+1, ap+2, , a2p+1 i rmn neaezate pe nici un taler
monezile a2p+2, a2p+3, , a3p+1.
Dac s<d, atunci avem de rezolvat problema P(3, n-1).
Dac s=d, atunci avem de rezolvat problema P(2, n-1).
Dac s>d, atunci avem de rezolvat problema P(4, n-1).
Problema P(3, n):
Notm p=(3n-1-1)/2 i observm c |B|=3p+1 i |C|=3p+2..
Aezm pe talerul stng monezile b1, b2, , bp i c1, c2, , c p+1, pe talerul
drept monezile bp+1, bp+2, , b2p i cp+2, cp+3, , c2p+2 i rmn neaezate pe
nici un taler monezile b2p+1, b2p+2, , b3p+1 i c2p+3, c2p+4, , c3p+2 .
Dac s<d, atunci avem de rezolvat problema P(3, n-1).
Dac s=d, atunci avem de rezolvat problema P(4, n-1).
Dac s>d, atunci avem de rezolvat problema P(3, n-1).
Problema P(4, n):
Notm p=(3n-1-1)/2 i observm c |B|=3p+2 i |C|=3p+1..
Aezm pe talerul stng monezile c1, c2, , cp i b1, b2, , bp+1, pe talerul
drept monezile cp+1, cp+2, , c2p i bp+2, b p+3, , b2p+2 i rmn neaezate pe
nici un taler monezile c2p+1, c2p+2, , c3p+1 i b2p+3, b2p+4, , b3p+2 .
Dac s<d, atunci avem de rezolvat problema P(4, n-1).
Dac s=d, atunci avem de rezolvat problema P(3, n-1).
Dac s>d, atunci avem de rezolvat problema P(4, n-1).
Problema P(5, n):
Notm p=(3n-1-1)/2 i observm c |B|=3p+1 i |C|=3p+1..
Aezm pe talerul stng monezile b1, b2, , bp , c1, c2, , cp i o moned
bun d1 , pe talerul drept monezile bp+1, bp+2, , b2p+1 i
cp+1, cp+2, , c2p i rmn neaezate pe nici un taler monezile b2p+2, b2p+3, ,
b3p+1 i c2p+1, c2p+4, , c 3p+1 .
Dac s<d, atunci avem de rezolvat problema P(5, n-1).
Dac s=d, atunci avem de rezolvat problema P(3, n-1).
Dac s>d, atunci avem de rezolvat problema P(4, n-1).
Observaie: dup fiecare cntrire se va face o renumerotare a monezilor n
fiecare din mulimile A, B, C, D.
Mai rmn de rezolvat cazurile particulare P(1, 1), P(2, 1), P(3, 1), P(4, 1),
P(5, 1). Fiecare din aceste cazuri se rezolv prin cel mult o cntrire.
Cazul P(1, 1) : deoarece avem 0 (zero) monezi considerm c este suficient
o cntrire pentru a "rezolva" problema (de fapt nu este nevoie de nici o
cntrire);
Cazul P(2, 1) : Avem monezile a1, d1. Aezm pe talerul stng moneda a1 i
pe talerul drept moneda d1.
Dac s<d, atunci moneda cutat este a1 i este mai uoar;
Dac s>d, atunci moneda cutat este a1 i este mai grea.

-6-

NOTE MATEMATICE
Nu putem avea s=d.
Cazul P(3, 1) : Avem monezile b1,c1,c2. Aezm pe talerul stng moneda c1 i
pe talerul drept moneda c2.
Dac s<d, atunci c2 este moneda cutat i este mai grea.
Dac s=d, atunci b1 este moneda cutat i este mai uoar.
Dac s>d, atunci c1 este moneda cutat i este mai grea.
Cazul P(4, 1) : Avem monezile b1,b2,c1. Aezm pe talerul stng moneda b1
i pe talerul drept moneda b2.
Dac s<d, atunci b1 este moneda cutat i este mai uoar.
Dac s=d, atunci c1 este moneda cutat i este mai grea.
Dac s>d, atunci b2 este moneda cutat i este mai uoar.
Cazul P(5, 1) : Avem monezile b1,c1,d1. Aezm pe talerul stng moneda b1
i pe talerul drept moneda d1.
Dac s<d, atunci b1 este moneda cutat i este mai uoar.
Dac s=d, atunci c1 este moneda cutat i este mai grea.
Nu putem avea situaia s>d.
Se observ c:
- numrul de cntriri suficient pentru fiecare din cele cinci probleme
este n.
- problema celor dousprezece monezi este de fapt problema P(1, 3).

SSM
H

Se pune acum problema determinrii numrului de cntriri necesar


(minim) rezolvrii acestor probleme. Partea care urmeaz aduce ceva nou i
deosebit n legtur cu aceast faimoas problem. Este mai dificil a gsi
numrul necesar de cntriri, dar este totui posibil. Pentru aceasta vom
folosi noiunea de entropie informaional (sau cantitate de informaie) a
unui experiment , noiune definit de ctre Claude Shannon n anul 1948.
Dac notm cu A un experiment i considerm p1,p2,,pn probabilitile
evenimentelor elementare corespunztoare experimentului A, atunci
formula de calcul a entropiei este
H(A) = H(p1,p2,,pn) = - (p1log2p1+p2log2p2++pnlog2pn) .
Dei aceast noiune are multe proprieti interesante noi vom enuna doar
dou din ele:
- fiind date n experimente oarecare A1, A2,, An avem
H(A1,A2,An) H(A1)+H(A2)++H(An)
egalitatea avnd loc atunci cnd experimentele sunt independente.
H(A) = H(p1,p2,,p n) este maxim atunci cnd probabilitile de
realizare ale evenimentelor sunt egale.
S revenim la problemele noastre.
Considerm un ir de k experimente auxiliare X1,X2,,Xk , care constau
fiecare din cte o cntrire cu balana. Deoarece fiecrui experiment auxiliar
i corespund trei evenimente elementare s<d, s=d sau s>d, vom avea
oricare ar fi 1ik relatia
H(Xi) - (1/3) (log2(1/3)+log2(1/3)+log2(1/3)) = log23 .
Entropia experimentului constituit din ntreg lanul de k experimente are
entropia
H(X1,X2,Xk)H(X1)+H(X2)++H(Xk) klog23 .

-7-

NOTE MATEMATICE
Problema P(1, n) cu n2:
Experimentul pentru care msurm cantitatea de informaie l notm cu X
i este acela de a afla moneda nestandard. Evenimentele elementare ce
compun experimentul X sunt n numr de 2(3n-3)/2 = 3n-3 i specific
faptul c oricare din cele (3n-3)/2 monezi poate fi cea cutat i faptul c
poate fi mai uoar sau mai grea. Probabilitatea ca o moned s fie cea
cutat i s fie mai uoar este 1/(3n-3), respectiv s fie mai grea este
1/(3n-3).
H(X) = H(p1,p2,,p2n) = - (1/(3n-3)) (log2(1/(3n-3))+ log2(1/(3n-3))++
log2(1/(3n-3))) = log2(3n-3).
Se pune ntrebarea "ct de mare trebuie s fie k pentru ca lanul de cntriri
X1,X2,,Xk s dea o cantitate de informaie mai mare sau egal cu a
experimentului X?".
Trebuie s rezolvm inegalitatea H(X) H(X1,X2,Xk) adic log 2(3n-3)
klog23 i obinem klog3(3n-3). Cea mai mic valoare posibil pentru k este
1+[log3(3n-3)] = 1+n-1 = n, deoarece 3n-1<3n-3<3n i deci n-1<log3(3n-3)<n.
Deci numrul minim de cntriri trebuie s fie n.
Problema P(2, n) cu n1:
Experimentul pentru care msurm cantitatea de informaie l notm cu X
i este acela de a afla moneda nestandard. Evenimentele elementare ce
compun experimentul X sunt n numr de 2(3n-1)/2 = 3n-1 i specific
faptul c oricare din cele (3n-1)/2 monezi poate fi cea cutat i faptul c
poate fi mai uoar sau mai grea. Probabilitatea ca o moned s fie cea
cutat i s fie mai uoar este 1/(3n-1), respectiv s fie mai grea este
1/(3n-1).
H(X) = H(p1,p2,,p2n) = - (1/(3n-1)) (log2(1/(3n-1))+ log2(1/(3n-1))++
log2(1/(3n-1))) = log2(3n-1).
Se pune ntrebarea "ct de mare trebuie s fie k pentru ca lanul de cntriri
X1,X2,,Xk s dea o cantitate de informaie mai mare sau egal cu a
experimentului X?".
Trebuie s rezolvm inegalitatea H(X) H(X1,X2,Xk) adic log 2(3n-1)
klog23 i obinem klog3(3n-1). Cea mai mic valoare posibil pentru k este
1+[log3(3n-1)] = 1+n-1 = n, deoarece 3n-1<3n-1<3n i deci n-1<log3(3n-1)<n.
Deci numrul minim de cntriri trebuie s fie n.
Problemele P(3, n) si P(4, n) cu n1:
Experimentul pentru care msurm cantitatea de informaie l notm cu X
i este acela de a afla moneda nestandard. Evenimentele elementare ce
compun experimentul X sunt n numr de (3n-1)/2+(3n+1)/2 = 3n i specific
faptul c oricare din cele 3n monezi poate fi cea cutat i faptul c poate fi
mai uoar sau mai grea. Pentru o moned probabilitatea ca ea s fie cea
cutat (ea va fi evident mai uoar dac este din mulimea B - nu se poate
altfel - i va fi mai grea dac este din C) este 1/3n.
H(X) = H(p1,p2,,p2n) = - (1/3n) (log2(1/3n)+ log2(1/3 n)++ log2(1/3n)) =
log23n.
Se pune ntrebarea "ct de mare trebuie s fie k pentru ca lanul de cntriri
X1,X2,,Xk s dea o cantitate de informaie mai mare sau egal cu a
experimentului X?".

-8-

NOTE MATEMATICE
Trebuie s rezolvm inegalitatea H(X) H(X1,X2,Xk) adic log 23n klog23 i
obinem kn. Deci numrul minim de cntriri trebuie s fie n.

SSM
H

Problema P(5, n) cu n1:


Experimentul pentru care msurm cantitatea de informaie l notm cu X
i este acela de a afla moneda nestandard. Evenimentele elementare ce
compun experimentul X sunt n numr de (3n-1)/2+(3n-1)/2 = 3n 1 i
specific faptul c oricare din cele 3n-1 monezi poate fi cea cutat i faptul
c poate fi mai uoar sau mai grea. Pentru o moned probabilitatea ca ea
s fie cea cutat (ea va fi evident mai uoar dac este din mulimea B - nu
se poate altfel - i va fi mai grea dac este din C) este 1/(3n-1).
H(X) = H(p1,p2,,p2n) = - (1/(3n-1)) (log2(1/(3n-1))+ log2(1/(3n-1))++
log2(1/(3n-1))) = log2(3n-1).
Se pune ntrebarea "ct de mare trebuie s fie k pentru ca lanul de cntriri
X1,X2,,Xk s dea o cantitate de informaie mai mare sau egal cu a
experimentului X?".
Trebuie s rezolvm inegalitatea H(X) H(X1,X2,Xk) adic log 2(3n-1)
klog23 i obinem klog3(3n-1). Deoarece
3n-1<3n-1<3n vom avea n-1<log3(3n-1)<n. Deci numrul minim (necesar) de
cntriri trebuie s fie n.
Am demonstrat aadar c numrul de cntriri necesar i suficient pentru
rezolvarea oricreia din cele cinci probleme este n .
Probleme propuse :
1. Se dau dousprezece monezi dintre care unsprezece au aceeai greutate
standard i una are greutatea mai mic. Care este numrul de cntriri
necesar i suficient pentru a afla moneda mai uoar folosind o balan.
2. Se dau n monezi dintre care n-1 au aceeai greutate i una are greutatea
mai mic. Care este cea mai mare valoare pentru n astfel nct s fie
posibil determinare monezi mai uoare din trei cntriri cu o balan.
3. ntr-o intreprindere sunt patru maini care confecioneaz monezi
identice. Se tie c una din ele s-a stricat i produce monezi diferite ca
greutate. Cu ajutorul unei balane s se determine cu cel mult dou
cntriri maina care s-a defectat i s se specifice dac monezile
produse de ea sunt mai grele sau mai uoare.

Bibliografie:
Silviu Guiau, Radu Theodorescu
Matematica i informaia Editura tiinific Bucureti 1965

-9-

NOTE MATEMATICE
Funciile spline ca abordare a analizei numerice elementare

Adela Ionescu
Univ. din Craiova
Colegiul Univ. Dr. Tr. Severin

I. INTRODUCERE.
Prin analiza numerica elementara intelegem metodele, al caror pionier
a fost Newton, de obtinere a unor aproximari numerice pentru interpolare,
integrare, diferentiere, si alte astfel de functionale, de functii de o variabila.
In aceste metode, interpolarea joaca un rol privilegiat, chiar fundamental.
Incepand din 1949, Arthur Sard a introdus un nou punct de vedere care a
condus la formulele sale de cea mai buna aproximare. In aceasta noua
abordare a vechilor metode ale lui Newton, parea ca interploarea si-ar pierde
rolul sau fundamental, pe care il avea in opera lui Newton. Insa, daca
interpolarea polinomiala a datelor se inlocuieste cu interpolarea prin asanumitele functii spline naturale, interpolarii i se restituie rolul sau central.
II.O ABORDARE NOUA A ANALIZEI NUMERICE ELEMENTARE.
Aceasta noua abordare are mult de a face cu un tip de functii foarte
elementare, functiile spline. Dupa cum se stie, cele mai simple functii, in
matematici pure si aplicate, sunt polinoamele. Fie k clasa polinoamelor de
grad care nu depaseste k. Intai exista 0, constantele. Apoi, 1, adica liniile
drepte, si apoi 2, parabolele. Cu graficele si problemele aferente, aceste este
cel mai inalt subiect din matematica scalara. Apoi vin cubicele, si mai
interesante, etc. Cand mentionam polinoame, trebuie amintita teorema lui
Weierstrass, care afirma ca polinoamele aproximeaza, cu orice eroare
prescrisa, orice fel de curba continua definita intr-un interval compact.
Dupa polinoame, exista functiile in scara, care sunt strans legate de
0. Daca luam integrala unei functii in scara obtinem o functie liniara
franta. Daca mai integram odata, obtinem o succesiune de arce de parabola.
Ordinul de continuitate creste cu fiecare integrare: am pornit cu o functie cu
discontinuitati; dupa o integrare obtinem o functie continua, si dupa doua
integrari, o functie cu derivata continua. Acestea sunt deja functii spline. In
general, printr-o functie spline Sm se intelege o succesiune de arce
polinomiale, de grad care nu depaseste m, care sunt unite in unele puncte
numite noduri, in care ordinul de continuitate este atat de inalt cat poate fi
fara ca cele doua arce sa fie parti ale aceluiasi polinom. Altfel spus, o functie
spline Sm difera de un polinom Pm de grad m prin aceea ca, desi ambele au
m derivate, in timp ce a m-a derivata a unui polinom este constanta, a m-a
derivata a unui spline este o functie in scara.

III. INTERPOLARE SPLINE NATURALA.

Sa consideram mai intai n puncte


(1 ) x1 < x 2 <K < x n ( n 1) ,
si un intreg m asa incat 1 m < n. Printr-un spline natural de grad 2*m-1,
cu nodurile (1), se intelege o functie S(x) cu urmatoarele trei proprietati:

- 10 -

NOTE MATEMATICE
( 2) S ( x ) 2*m1 in fiecare int erval ( x i , x i +1 ) , i = 1,2, K , n 1.
(3) S ( x ) C 2*m 2 ,
.
( 4) S ( x ) m1 in ( , x1 ) si de asemenea in ( x n , )

SSM
H

Sa notam clasa acestor spline prin


(5) 2*m1 ( x1 , K , x n ) .
Observam ca 2*m-1 2*m-1(xi).
Prezentam mai jos cateva exemple simple.
Fie m=1. Conditiile (2), (3), (4), arata ca graficul lui S(x) este o linie
poligonala continua, cu varfuri in nodurile (1), si este asa incat S(x)=S(x1),
daca x < x1, si S(x)=S(x n), daca x > xn.
Fie m=2. Din nou conditiile arata ca S(x) este un spline cubic, deci cu
derivate prime si secunde continue, cu nodurile (1) si proprietatea
particulara ca cele doua ramuri infinite ale graficului sunt doua linii drepte.
Exista o reprezentare analitica a lui S(x). Sa consideram cazul m=2,
deci sa presupunem ca
( 6) S ( x ) 3 ( x 1 , x 2 , K , x n ) .
Din prima conditie (4), obtinem reprezentarea
( 7) S ( x ) = a 0 + a1 + c1 * ( x x1 ) + 3 + K + c n * ( x x n ) + 3 ,
unde trebuie notata ca a doua conditie (4) inca nu a fost pusa. Pentru a o
introduce si pe ea, sa consideram in primul rand x > x n si sa observam ca
putem renunta la indicele + in (4). Dezvoltand binoamele si grupand
termenii, obtinem ca S(x) va fi liniar in (xn, ) daca vom avea:
( 8) c1 + c 2 + K + c n = 0, c1 * x1 + c 2 * x 2 + K + c n * x n = 0 .
Numarand parametrii in relatia de mai sus, vedem ca (7) depinde de n+2
parametri, doi din ei putand fi eliminati prin ecuatiile (8). Astfel, putem
concluziona ca, daca S(x) 2*m-1(xi), atunci
( 9) S ( x ) depinde de n parametri liniari .
In aceste conditii, se poate formula afirmatia ca S(x) poate interpola in mod
unic n ordonate date in noduri. Aceasta se poate enunta in urmatoarea
teorema [2]:
Teorema: Sa presupunem ca 1m<n. Date numerele reale
(10) y 1 , y 2 , K , y n ,
exista un unic spline
(11) S ( x ) 2*m1( x1 , K , x n ),
asa incat
S ( xi ) = yi , i = 1,2, K , n.
Metoda de interpolare fiind liniara, putem considera functiile fundamentale
Li(x) definite prin
1 daca i = j,
(12) Li ( x ) 2*m1( x i ), Li x j =
0 daca i j
Aceste relatii ne permit sa scriem interpolantul (11) sub forma:

( )

(13) S ( x ) =

f ( x ) * L ( x ).
i

Daca functia f are proprietatea ca

- 11 -

NOTE MATEMATICE
(14) f ( x )

2 * m 1

(x ) ,
i

atunci unicitatea din teorema 1 implica faptul ca S(x)=f(x), pentru orice x.


Iar daca (14) nu este valabila, atunci putem scrie

(15) f ( x ) =

f ( x ) * L ( x) + R
i

( x) ,

cu un rest Rf(x). Relatia (15) se numeste formula de interpolare spline


naturala. - prescurtat F.I.S.N.
APLICATIE.
Pentru a ilustra cum functioneaza F.I.S.N. (15), sa consideram un exemplu
cu n=3, x1=-1, x2=0, x3=1; sa alegem m=2. Se pot calcula explicit expresiile
pentru cele trei functii fundamentale Li(x) (i=1,2,3) in (15), folosind definitia
lor (12).
Din definitia (17) a functiilor Li(x) avem ca
f ( x) = f ( x1 ) * L1( x) + f ( x2 ) * L2 ( x) + f ( x3 ) * L3 ( x) ,
unde am considerat Rf(x)=0, cu f 2*m-1(x).
Aplicand relatia (8) pentru functia
S ( x) = a0 * x + a1 + c1 * ( x x1) + 3 + c2 * ( x x2 ) + 3 + c3 * ( x x3 ) + 3 ,
in nodurile -1,0, si 1 specificate, obtinem un sistem de ecuatii in
necunoscutele c1, c2, c3 din care reiese setul de solutii (c1, -2*c1, c1). Atunci
obtinem
S ( x) = a0 * x + a1 + c1 * ( x x1) + c1 * ( x x3 ) ,
renuntand deja la indicele +.
Intrucat f 2*m-1(x), avem conform celor afirmate mai sus, ca f(x)=S(x), si
atunci obtinem
3

S ( x ) = a0 * x + a1 + c1 * ( 2 * x 3 + 6 * x 2 ) = S ( 1) * L1( x ) + a1 * L2 ( x ) + S (1) * L3 ( x ) .
Efectuand calculele, egaland membrul stang cu cel drept dupa a0, a1 si c1,
obtinem in final expresiile
1
L1( x) = * ( x 3 + 3 * x 2 4 * x),
6
1
1
L2 ( x) = * x3 x 2 * x + 1, .
3
3
1
2
1
L3( x) = * x3 + * x 2 + * x
6
3
3

REFERINTE BIBLIOGRAFICE
[1] P.J.Davis, Interpolation and Approximation, Blaisdell, New York 1963.
[2] I.J.Schoenberg, Privelisti matematice, Editura Tehnica, Bucuresti,
1989.

- 12 -

NOTE MATEMATICE
O metod de rezolvare a ecuaiei de gradul al treilea

SSM
H

prof. Prenenu Vasile Doru

n aceast not abordm n prim instan rezolvarea sistemului:


u+v = a
ux + vy = b

(S)
2
2
ux + vy = c
ux3 + vy 3 = d
Rezolvare:
Notm cu S = x + y, P = xy i obinem un sistem cu necunoscutele P i
S astfel: nmulim ecuaia a dou cu x + y = S i obinem ux2 + vxy + uxy +
vy2 = bS . Folosind ecuaiile ( 1 ) i ( 3 ) obinem :
bS aP = c
(i)
nmulind ecuaia a treia cu x + y = S obinem: ux3 + ux2y + vxy2 + vy3 =cS
Folosind ecuaiile ( 2 ) i ( 4 ) obinem :
cS bP = d
( ii )
bS aP = c
Prin urmare am obinut sistemul format din ( i ) i ( ii ) :
cS bP = d
din care obinem soluia ( S, P). Revenind la notaia iniial S = x + y, P = xy
obinem soluiile x1, x2 i y1, y2 ale sistemului simetric care, apoi introduse
n primele dou ecuaii ale sistemului, conduc la soluiile u1, u2 i v1, v 2
corespunztoare .

Rezolvarea ecuaiei de gradul al treilea folosind sistemul ( S )


Pentru ecuaia ax3 + bx2 + cx + d = 0, a, b, c R, a 0 se cunoate c
are cel puin o soluie real. Pentru a rezolva ecuaia dat folosind sistemul
(S) scriem:
ax3 + bx2 + cx + d = (x + u)3 + (x + v)3
( iii ) ,
unde ,u, i v se obin prin metoda coeficienilor nedeterminai rezultnd
+ = a, u + v =b/3, u2 + v 2 =c/3, u3 + v2 = d adic sistemul (S).
Ecuaia ( iii ) devine :
[ 3 (x + u) + 3 (x + v)][ 3 (x + u)2 - 3 (x + u)(x + v) + 3 (x + v)2] = 0
cu rdcinile, x1 =

( u +
3

+3

) R

iar x2 i x3 sunt date de ecuaia de

gradul al doilea:

(3 2 3 + 3 2 ) x 2 + [23 2 u + 23 2 v 3 (u + v)]x + 3 2 u 2 + 3 2 v 2 3 uv = 0

- 13 -

NOTE MATEMATICE
Exemplu :
1.Considerm ecuaia de gradul al doilea:
2x3 + 3( 2 - 3 )x2 + 15x + 2 2 - 3 3 = 0
u + v = 2
ux + vy = 2 3

Scriem sub forma ( iii ) i obinem sistemul: 2


2
ux + vy = 5
ux 3 + vy 3 = 2 2 3 3
Rezolvnd sistemul prin metoda prezentat gsim:
(,,u,v) = (1,1, 2 ,- 3 )i (1,1,- 3 , 2 ) i ( iii )devine :
2x3 + 3( 2 - 3 )x2 +15x + 2 2 - 3 3 = ( x + 2 )3 + ( x - 3 )3
(x + 2 )3 + (x - 3 )3 = 0
(x + 2 + x - 3 ) [(x + 2 )2 (x + 2 )(x - 3 ) + (x - 3 )2] = 0
(2x +

2-

3 ) i x2,3 = - (- 2 +

3 + 15 + 6 6 ).

2. Lsm cititorului rezolvarea ecuaiei: x3 6x2 + 15x 14 = 0. Prin metoda


indicat mai sus solutiile sunt: x1 = 2, x2,3 = 2 + 3 .

****************
Quelques chiffres

prof. Grecu Cornelia


Vous avez contemple des nuits aussi belles que cette nuit de Grece que
decrit si poetiquement Chateaubriand. Mais vous doutiez-vous que cette
magnifique lune qui semble immobile se deplase en realite a la vitesse dun
kilometre a la seconde, soit 3600 km a lheure ? Et quainsi elle est
beaucoup plus rapide encore quun avion a reaction ? Saviez-vous que la
lumiere scintillante de letoile polaire met 36 ans pour parvenir jusqu'
nous ? Mais ne croyez pas que cette etoile soit la plus lointaine. On a pu
capter avec des appareils ultramodernes le bruit occasionne par la rencontre
detoiles situees au-dela de la voie lactee, dans la constellation du Cygne. Ce
bruit avait mis (a la vitesse du son) plusieurs milliards dannees pour nous
parvenir ! La terre fait partie dun univers comprenant cent milliards
detoiles. Cet Univers tourne sur lui-meme a al vitesse de 1600000 km/h.
Certaines de ces etoiles (dites jeunes ) ont lintensite de cent mille soleils.
Nous ne voyons a lil nu quune partie de ce systeme infini. Celle qui nous
est la plus familiere est lEtoile Polaire. Elle se trouve a 90 annees-lumiere,
ce qui equivaut a dix mille milliards de kilometres multiplie par 90 !

- 14 -

CERCUL DE MATEMATICA

SSM
H

Tem pentru grupele de performan, clasa a V-a


Numere raionale pozitive
Prof. Rodica Popescu
Prof. Daniel Stretcu
D1. O pereche de numere naturale m i n (n 0), scrise sub forma

m
, se
n

numete fracie.
m p
m
p
D2. Fraciile
i
sunt echivalente ~ dac mq = np.
n
q
n q
m
D3. Un numr raional este mulimea tuturor fraciilor
echivalente cu
n
a
fracia dat .
b
a
Dac notm A numrul raional definit mai sus, atunci fracia
este un
b
reprezentant al numrului raional A. Orice fracie din mulimea care
definete un numr raional poate fi un reprezentant al acestui numr
m

raional. Mulimea numerelor raionale pozitive este Q+ = | m, n N ; n 0


n

Aplicaii
1. S se demonstreze c fracia
Soluie Fie d= (21n+4,14n+3)
d|2(21n+4)
d|3(14n+3)
d|(42n+9) (42n+8)
fracia dat este ireductibil.

21n + 4
este ireductibil ( ) nN.
14n + 3

d|21n+4
d|14n+3

d|42n+8
d|42n+9

d|1
d=1

2. a) S se gseasc toate numerele nN, pentru care numrul


b) S se demonstreze c pentru orice nN, numrul

2n 1
N.
7

2n + 1
N.
7

Soluie a.
I. Fie n multiplu de 3, adic putem scrie n = 3k i 2n-1 = 23k-1 = 8k-1
Diferena a dou puteri cu acelai exponent se divide ns cu diferena
2n 1
bazelor, deci 8k-1 se divide cu 8-1=7

N deci nM3.
7
II. Fie n de forma n=3k+1
2n-1 = 23k+1-1 = 28k-1=2(7+1)k-1
k
dar (7+1) d prin mprirea la 7 restul

2(7+1)k d prin mprirea la 7 restul 2

2n-1 = 2(7+1)k-1 d prin mprirea la 7 restul 1


2n 1

pentru n=3k+1 numrul


N.
7
III. Fie n de forma n=3k+2
2n-1 = 23k+2-1 = 48k-1=4(7+1)k-1

prin mprirea la 7 se obine restul 3

- 15 -

CERCUL DE MATEMATICA
2n 1
N.
7
n concluzie, 2n-1 se divide cu 7 dac i numai dac n este multiplu de 3
Soluie b.
Dac n=3k 2n=23k=7m+1 2n+1=7m+2 deci nu se divide cu 7.
Dac n=3k+1 2n+1=2(7m+1)+1=14m+3 deci nu se divide cu 7.
Dac n=3k+2 2n+1=4(7m+1)+1=28m+5 deci nu se divide cu 7.
n concluzie, pentru nici un numr nN, 2n+1 nu se divide cu 7.
69
3. S se determine numerele de forma xyz cu proprietatea c 3
N .
x + y2 + z
69
Soluie 3
N

x3 + y2 + z {1, 3, 23, 69}.


x + y2 + z
a)
x3 + y2 + z = 1
x = 1, y = 0, z = 0
3
2
b)
x +y +z=3
x = y = z = 1 sau x = 1, y = 0, z = 2
c)
x3 + y2 + z = 23
x<3
x{1, 2}
2
x=1
y + z = 22
y = 4, z = 6
x=2
y2 + z = 15
y = 3, z = 6
3
2
d)
x + y + z = 69
x<5
x{1, 2, 3, 4}
x=1
y2 + z = 68
y = 8, z = 4
x=2
y2 + z = 61
nu are soluii
x=3
y2 + z = 42
y = 6, z = 6
2
x=4
y +z=5

y = 1, z = 4 sau y = 2, z = 1
xyz {100, 102, 111, 146, 184, 236, 366, 414, 421}

pentru n=3k+2 numrul

4. S se gseasc numerele ab pentru care

ba
N.
9a + b

ba
N 9a+b| ba 10b+a=k(9a+b), kN*
9a + b
10b + a
89a
9a+b|89a dar 9a + b 90 < 289 , 9a+b >a ,
k=
= 10
9a + b
9a + b
89 este prim 9a+b = 89 deci a=9 i b=8.
PROBLEME PROPUSE
1 2
n
1
1
1
1
5. Fie numerele A = + + ... +
i B = 1 + 2 + 3 + ... + n
, nN*.
2 3
n +1
2
3
4
n +1
Artai c numrul A+B-2n
este natural.
1 1
4
6. Folosind inegalitatea + f
, ( ) a, b f 0, a b , artai c:
a b a+b
1
1
1
13
+
+ ... +
f
501 502
1000 20
1
1
1
7. Fie suma S = 2 + 2 + ... +
. S se arate c 0,098<S<0,112.
10 11
10002
8. Fie A = 11...1 . Artai c A se divide cu 41 n este multiplu de 5.

Soluie

n ori

Bibliografie:
A.Bluc, I.icale
Matematica pentru concursurile colare
A. Morozov, I. Petrakov Olimpiadele internaionale de matematic

- 16 -

CERCUL DE MATEMATICA
Tem pentru grupele de performan, clasa a VI-a

SSM
H

PTRATE PERFECTE
Catedra de matematic-coala 11-Dr.Tr. Severin
Prof.coordonator Victor Sceanu
n manualele colare i mai ales n revistele de matematic apar multe
probleme, ce cer s artm dac un numr este sau nu ptrat perfect.
n cele ce urmeaz ne propunem s redm unele din cele mai utile
metode de a arta c un numr este sau nu ptrat perfect .
Preliminarii
a) Spunem c numrul n este ptrat perfect dac exist un numr a astfel
nct n=a2
b) Dac un ptrat perfect n conine un factor prim p atunci, cu necesitate,
acest factor se va gsi n descompunerea lui n la o putere par
c) n sistemul zecimal avem scrierea:
anan-1an-2a2a1a0 = a n 10n+an-1 10n-1+an-2 10n-2++a2 102+a1 10 +a0
d) Fiind date numerele naturale a i b, b=0, exist, n mod unic dou
numere naturale q i r astfel nct : a = b q+r cu r {0,1,2,b-1}
e) Avem identitatea, pentru orice n N
xn+1-1= (x-1)(xn+xn-1++x+1) sau
xn+xn-1++x+1= (x n+1-1)/(x-1).
Pentru a arta c un numr este sau nu ptrat perfect redm n
continuare urmtoarele metode:
I. Calculul ultimei cifre a numrului
Notm cu u(n) ultima cifr a numrului n
Din: u(0)= 0; u(1 2)=1; u(22)=4; u(32)=9; u(42)=6; u(52)=5; u(62)=6; u(72)=9;
u(82)=4; u(92)=1 deducem c ultima cifr a unui ptrat perfect nu poate fi
dect una din cifrele: 0;1;4;5;6;9.
De aici deducem c un ptrat perfect nu poate avea ca ultima cifr una din
cifrele : 2,3,7,8.
Observaie. Nu orice numr care are ultima cifr una din cifrele 0, 1,
4, 5, 6, 9 este ptrat perfect, deci condiia este numai necesar dar nu i
suficient.
Exemple
1) Artai c:
n = 1+2 3+4 5 6+7 8 9 10+11 12 13 14 15 +nu poate fi ptrat perfect.
Soluie: Avem c u (1+2 3)=7, iar ultima cifr a restului de termeni este
0, deci u(n)=7+0=7 i nu este ptrat perfect.
2) Artai c numrul: 32003-20032 nu este ptrat perfect.
Soluie: Avem u (32003) =u (3 4 500+3)= u [(34)500 33]= u (81500 27 ) =u (1 27)
=7, iar u (20032) = 9 i atunci u(3 2003-20032)= u (17-9)= 8, deci nu poate
fi ptrat perfect
3) S se arate c numerele de forma 2000 n+2003, nu sunt ptrate perfecte,
oricare ar fi n N
Soluie: Avem u (2000 n)= 0 i u (2003)=3, deci u (2000.n +2003)=0+3=3
i nu poate fi ptrat perfect

- 17 -

CERCUL DE MATEMATICA
4) S se determine toate numerele naturale nenule n pentru
care:1!+2!+3!++n! este ptrat perfect (k!=1 2 3 k)
Soluie: Pentru n =1 avem 1!= 1 care este ptrat perfect
Pentru n =2 avem 1!+2!=1+1 2=3 care nu este ptrat perfect
Pentru n =3 avem 1!+2!+3!=1+2+6=9 care este ptrat perfect
Pentru n >4 avem 1!+2!+3!+4!=33, iar 5!+6!++n! este un numr care se
termin n cifra 0, rezult c pentru n>4, 1!+2!+3!+4!++n! se termin n
cifra 3 i deci nu este ptrat perfect. Deci valorile lui n sunt 1 i 3.
5) S se determine valorile lui n N* pentru care numrul a= 2n!+1 nu este
ptrat perfect. (n!= 1 2 3 n)
Soluie: Pentru ca a s nu fie ptrat perfect trebuie s aib ultima cifr
:2;3;7 sau 8. Cum puterile lui 2 au ultima cifr 2;4;6 sau 8 trebuie ca 2n!
s aib ultima cifr 2 sau 6, deci n! s fie M4+1 sau M4.
Cum n! =123n, rezult c n! poate fi M4+1 numai pentru n=1 i
atunci a= 1 2 3 n, rezult c n! poate fi M4+1 numai pentru n=1 i
atunci a=21!+1=2+1=3 care nu este ptrat perfect.
Pentru ca n ! s fie M4trebuie ca n >4. Deci n {1,4,5,6 }
II. Formele sub care poate fi scris un ptrat perfect
Ptratul oricrui numr natural poate fi de forma 4K sau 4K+1
Demonstraie: Dac n N este par, atunci n =2p i deci n2=4p2=4K. Dac n
N este impar, atunci n = 2p+1 i deci n2=(2p+1)2=4p2+4p+1 =4(p+1)+1=4k+1
Ptratul unui numr natural impar poate fi i de forma 8k+1
Demonstraie: Din n
N i impar, rezult n=2p+1 de unde
n2=(2p+1)2=4p2+4p+1 =4p(p+1)+1 i cum p i p+1 sunt consecutive, rezult
p(p+1)=M2 rezult n2=4 2k+1=8k+1
Observaie Forma 8k+1 este mai slab ca forma 4p+1 deoarece 8k+1=4
2k+1=4p+1
Exemple
1) Suma ptratelor a patru numere naturale nenule consecutive nu poate fi
ptrat perfect.
Soluie: Avem: n N* i atunci (n-1)2+n2+ (n+1)2+ (n+2)2=n2-2n +1+n2+
n2+2n+1+ n2+4n+4=4n2+4n+6=4n2+4n+4+2= 4(n2+n+1)+2= 4k+2
Dar nici un ptrat perfect nu este de forma 4k+2, k N.
2) Fie n N, impar. Artai c numrul a = 3n+5n+3 nu este ptrat perfect.
Soluie: Avem : a =(4-1)n+ (4+1)n+3 = M4-1+M4+1+3= M4+3 = 4k+3. Dar
nici un ptrat perfect nu este de forma 4k+3.
3) Numrul 2004n+3 nu poate fi ptrat perfect oricare ar fi n N
Soluie: 2004=M4 rezult 2004n= M4 2004n+3= M4+3= 4k+3 i nu este
ptrat perfect.
III. Folosirea formulelor de calcul prescurtat
Se arat c un numr este ptrat perfect dac poate fi pus sub forma (A+B)2,
sau se ncadreaz ntre ptratele a dou numere consecutive pentru a arta
c nu este ptrat perfect.

- 18 -

CERCUL DE MATEMATICA
Exemple
1) Fie numerele A= 1111 avnd 2n cifre i B = 4444 avnd n cifre scrise
n sistenmul zecimal. Demonstrai c A+B+1 este ptrat perfect.
Soluie: Avem
10 2n 1
A+B+1=( 102n-1+102n-2++10+1) +4 (10n-1+10n-2++10+1)+1=
+4
9
10 n 1
10 2 n + 410 n + 4
10 n + 2 2
+1=
=(
)
9
9
3

SSM
H

Cum 10n+2 = 1000+2=10002 are suma cifrelor 3, obinem c


10 n + 2
N
3
2) S se arate c numrul A = 11...155...5 6 este ptrat perfect.
n +1 ori

Soluie: Avem: A = 11...1 10


n +1 ori

A = (10 + 10
n

A=

n +1

n 1

n +1

n ori

+ 55...5 10 + 6 = 11...1 10 n +1 + 55...5+ 1


n +1 ori

n ori

+ ... + 10 + 1) 10

n +1

10 1 n +1
10 1
10
10 + 5
+1 =
10 1
10 1

10 2( n +1) + 4 10 n +1 + 4 10 n+1 + 2

A=
=
9
3

n +1

2 ( n +1)

n +1 ori

n 1

+ ... + 10 + 1) + 1

n +1

5+9

+ 5 (10 + 10
n

10

n +1

+ 5 10
9

3) S se demonstreze c oricare ar fi n N*, n2+n nu poate fi ptrat perfect.


Soluie
Avem c n2<n2+n< n2+2n+1, deci n2<n2+n<(n+1)2
Cum n2 i (n+1)2 sunt dou ptrate consecutive i cum ntre ele nu exist
nici un ptrat perfect, rezult c n2+n nu este ptrat perfect.
4) Artai c produsul a patru numere naturale consecutive, nu poate fi
ptrat perfect, dac primul numr este nenul.
Soluie
Avem:
n(n+1)(n+2)(n+3)=
(n2+3n)(n2+3n+2)=
(n2+3n+1-1)
2
2
2
2
2
(n +3n+1+1)= (n +3n+1) -1<(n +3n+1)
Pe
de
alt
parte
avem
n(n+1)(n+2)(n+3)=
(n2+3n)(n2+3n+2)=
(n2+3n)2+2(n2+3n)> (n2+3n)2. Deci (n2+3n)2< n(n+1)(n+2)(n+3) <
(n2+3n+1)2 i cum (n2+3n)2 i (n2+3n+1)2 sunt ptrate consecutive,
rezult concluzia.

- 19 -

CERCUL DE MATEMATICA
Tem pentru grupele de performan, clasa a VII-a
Ecuaii diofantice ax + by + c = 0 , x 2 + y 2 = z 2
Prof. Roman Florentina Gabriela
1. Algoritmul lui Euclid
Daca avem dou numere a i b unde a,b Z , atunci putem face
mprirea lui a la b. facnd mprtirea cu rest gsim c pentru un anumit
k N , avem:
a = c1 b + r1
0 < r1 < b1 ,
b = c 2 r1 + r2
0 < r2 < r1 ,
r1 = c3 r2 + r3
0 < r3 < r2 ,
(1)
LLLLLLL
LLLLL
rk 2 = c k rk 1 + rk ,
0 < rk 1 < rk ,
rk 1 = c k +1 + rk ,
unde c1 ,...., c k +1 N * .
Acest ir de operaii se numete algoritmul lui Euclid. tim c ultimul rest
nenul este cel mai mare divizor comun al numerelor a i b.
2. Ecuaia ax+bz+c = 0 ,a,b,c Z

(2)

Lema 1: Ecuaia (2) are soluie in Z dac i numai dac d = (a, b) ) / c.


Lema 2: Dac (a, b ) = 1, iar ( x0 , y 0 ) este o soluie particular a ecuaiei (2),
atunci soluia general din Z a acestei ecuaii este dat de x = x0 kb i

y = y 0 + ka, cu k Z .
Corolar 3: Fie a, b, c Z a.. d = ( a, b ) / c, a = da ' , b = db ' , c = dc ' .
Dac ( x0 , y 0 ) Z Z este o soluie particular a ecuaiei a 'x +b ' y + c ' = 0, atunci
solutia general a ecuaie (2) este dat de x = x0 kb`, y = y 0 + ka ' cu k Z .
Observatie: innd cont de Lema 1 si Corolarul 3 deducem c atunci cnd
suntem pui n situaia de a rezolva o ecuaie de forma (2), n cazul n care
d = (a, b) c , este recomandabil s mparim ambii membri ai ecuaiei prin d,
transformnd-o astfel n ecuaia echivalent a ' x + b ' y + c ' = 0
a
b
c
(cu a ' = , b ' = , c ' = ) . Cum (a ' , b ' ) = 1, forma general a soluiilor ecuaiei
d
d
d
a ' x + b ' y + c ' = 0 este data de Lema 2.
Teorema 4: Dac a i b sunt dou numere intregi i d este cel mai mare
divizor comun al lor, atunci exist dou numere intregi i , astfel
d = a + b .
Dac avem o ecuaie de tipul (2) , putem aplica algoritmul lui Euclid.
Considerm dou
numere a, b Z , (a, b ) = 1 i dac de exemplu avem
a = c1 * b + r1
urmatorul algoritm al lui Euclid:

b = c2 * r1 + 1
r1 = c3 *1 + 0

- 20 -

CERCUL DE MATEMATICA
din Teorema 4 rezult c putem avea a + b = 1. Din algoritmul lui Euclid
1 = b c 2 r1
obinem c:
i din cele dou rezult c
r1 = a c1 b
1 = b c2 (a c1b) 1 = b c 2 a + c1c 2 b 1 = a (c 2 ) + ... + b(1 + c1c2 .
Exemplu: S se rezolve ecuaia: 5x+3z+7=0 . (*)
Soluie: Avem a = 5 , b = 3, c=7 i se observ c (a,b) = 1, astfel c
ecuaia (*) are soluiii in Z Z , conform Lemei 2.
Folosim algoritmul lui Euclid pentru a gsi soluia particular:
5 = 1 3 + 2

SSM
H

3 = 1 2 + 3
2 = 1 2 + 0.
Cu ajutorul Teoremei 4 avem 5 + 3 = 1 , iar pe 1 il putem scrie:
1 = 5 (1) + 3(1 + 1 1) 1 = 5 (1) + 3 2 = 1 si = 2 .
Ecuaia ( ) daca o imprim prin c devine:
x
y
x
y
5
+ 3
1 = 0 5 + 3 = 1 =
si =
x = ( 1) ( 7) si
7
7
7
7
y = 2 (7) x = 7 si y = 14
Soluia particular este (7,-14). Din corolarul 3 obtinem soluia general:
x = 7 3k si y = 14 + 5k .
3. Ecuaia x 2 + y 2 + z 2 (3)
n primul rnd trebuie observat c dac tripretul (x,y,z) de numere
intregi verific ecuaia (3), atunci aceeasi ecuaie va fi sadisfacut de orice
triplet de forma (x, y, z ) , unde si reciproc.
De aceea, pentru a gsi toate soluiile ecuaiei (3), constnd din numere
diferite de 0, este suficient s gsim soluiile (x,y,z) pentru care numerele
x,y,z sunt relativ prime (adic nu au nici un divizor prim diferit de 1).
Este clar c dac intr-o soluie (x,y,z) a ecuaiei (3) dou dintre numerele
x,y,z au un divizor comun = 1 atunci si cel de al treilea numr se divide
cu .
De aceea ne putem restrnge la soluiile ce constau din numerele relative
prime dou cte dou, pe care le vom numi soluii primitive.
Dac (x,y,z) este o soluie a lui (3), atunci in mod evident si (y,x,z) este o
soluie.
Pe de alt parte dac (x,y,z) este soluie, atunci x sau y este par (dac x si
y ar fi impare, atunci x 2 + y 2 ar fi de forma 4k+2, pe cand ptratul unui
numr intreg nu poate fi decat de forma 4k sau 4k+1).
n plus, este evident c dac (x,y,z) este soluie, atunci si ( x, y, z ) vor fi
soluii.
Lema 5. Orice soluie particular (x,y,z) de numere naturale (cu n par) a
ecuaiei (3) este de forma x = 2mn, ym 2 n 2 , z = m 2 + n 2 cu m,n , (n,m)=1, iar
m,n au pariti diferite.

- 21 -

CERCUL DE MATEMATICA
Corolar 6: Soluia general a ecuaiei (3) este
x = 2 rmn, yr ( m 2 n 2 ), z = r ( m 2 + n 2 ) cu r,m,n Z.
Exemplu: S se calculeze ecuaia
( x 533) 2 + ( y 533) 2 = 2 5332 . (*)
Solutie: Cum membrul drept din (*) este par deducem c dac
(x,y) , atunci x-533 i y-533 au aceeai paritate.
1
1
Astfel p = ( x + y) 533 si q = ( x y ) sunt numere intregi.
2
2
Deducem imediat ca x-533=p+q si y-533=p-q i cum
( p + q ) 2 + ( p q ) 2 = 2 p 2 + 2q 2
ecuaia (*) devine:
p 2 + q 2 = 5332
Observm c dac n = 5332 = 412 13 2 , atunci soluiile vor fi de forma
x=2mn, y = m 2 n 2 , z = m 2 n 2 .
Mai exist si ecuaii de tipul x 2 ay 2 = 1, a , care pot avea o infinitate
de soluii.
Exemplu: S se arate c ecuaia x 2 3 y 2 = 1 are o infinitate de soluii intregi.
Soluie: O soluie particular a ecuaiei este x=7, y=4.
Fie ( , ) o soluie a ecuaiei

2 3 2 = 1 4 6 2 2 + 9 4 = 1 ( 2 + 3 2 ) 2 3(2 ) 2 = 1 ( 2 + 3 2 ,2 )
este o alt soluie pentru ecuaia noastr.
Ecuaia x 2 3 y 2 = 1 are o infinitate de soluii intregi.
Probleme propuse:
1. S se arate c ecuaia : x 2 + 2 x = 2 y 2 are o infinitate de soluii intregi.
( Revista Alfa, 2003)
2. S se demonstreze ca exist o infinitate de solutii pentru care
3x 2 7 y 2 + 1 = 0
3. S se resolve ecuaia : ( x 989) 2 + ( y 989) 2 = z 2 cu z 2 = 2 989 2
4. S se resolve ecuaia: 3 x + 2 y + 4 = 0
5. S se resolve ecuaia: 7 x + 5 y + 9 = 0
Bibliografie:
1. D.Buneag, F.Boboc, D.Piciu
Elemente de arimetic i teoria numerelor - Ed.Radical Craiova-1998.
2. W.Sierpinsky
Elementary Theory of Numbers Polshi Academic Nauk Warsaw-1964.
3. G.Eckstein
Fracii continue RMT.nr.1,pp.17-36-1986.

- 22 -

CERCUL DE MATEMATICA

SSM
H

Tem pentru grupele de performan, clasa a VII-a


TEOREMA LUI PITAGORA GENERALIZAT
prof.Constantin Magdalena
prof.Draga Ttucu Porfirel

Noiuni teoretice
Teorema 1 (Teorema lui Pitagora generalizat):
Dac n triunghiul ABC, C este ascuit i D = prBC A atunci

AB 2 = AC 2 + BC 2 2 BC DC
Dem: a) B ascuit D (BC )

ABD dreptunghic AB 2 = AD 2 + BD 2
ADC dreptunghic AD 2 = AC 2 DC 2
BD = BC DC

AB 2 = AC 2 DC 2 + (BC DC )2 = AC 2 DC 2 + BC 2 + DC 2 2 BC DC =
= AC 2 + BC 2 2 BC DC
b) B obtuz B (DC )
2

A
2

ADB dreptunghic AB = AD + BD
ADC dreptunghic AD 2 = AC 2 DC 2
BD = DC BC

D B

AB 2 = AC 2 DC 2 + (DC BC )2 = AC 2 DC 2 + DC 2 2 DC BC + BC 2 =
= AC 2 + BC 2 2 BC DC
c) m B = 90 atunci relaia AB 2 = AC 2 + BC 2 2 BC DC rezult din
teorema lui Pitagora ( B=D ).
Teorema 1 Dac n triunghiul ABC, C este obtuz i D = prBC A atunci

()

AB 2 = AC 2 + BC 2 2 BC DC
Dem: C fiind obtuz C (BD )
2

ABD dreptunghic AB = AD + BD
ACD dreptunghic AD 2 = AC 2 CD 2
BD = BC + CD

CD

AB 2 = AC 2 CD 2 + ( BC + CD )2 = AC 2 CD 2 + BC 2 + CD 2 + 2 BC DC =

= AC 2 + BC 2 + 2BC CD

Aplicaii
Aplicaia 1 (Relaia lui Stewart)
Dac A, B, C sunt trei puncte coliniare cu B ( AC ) i O un punct exterior
dreptei AC, atunci are loc relaia: OA2 BC OB 2 AC + OC 2 AB = AB BC AC .
Rezolvare:
Fie D piciorul perpendicularei din O pe AC. Putem avea trei situaii: D (BA ,
D (BC , D = B . Primele dou cazuri sunt analoage, deci putem presupune
de exemplu c D (BA i deci B este ascuit n OBD .

- 23 -

CERCUL DE MATEMATICA
O

Aplicnd teorema lui Pitagora generalizat n OAB

OA2 = AB 2 + OB 2 2 AB BD
(1)
Deoarece n OBC avem B obtuz, aplicnd
teorema lui Pitagora generalizat
A
D
B
C
(2)
OC 2 = OB 2 + BC 2 + 2 BC DB
Amplificm relaia (1) cu BC i relaia (2) cu AB i adunndu-le membru cu
membru

OA2 BC + OC 2 AB = AB 2 BC + OB 2 BC 2 AB BD BC + OB 2 AB + BC 2 AB +
+ 2 BC DB AB

AB BC ( AB + BC ) + OB 2 ( AB + BC ) = AB BC AC + OB 2 AC , deci relaia lui


Stewart este adevrat.
Dac B=D, aceleai relaii sunt adevrate cu BD = 0.
Aplicaia 2
Aria unui triunghi ABC este dat de formula: S = p ( p a )( p b )( p c ) unde
2 p = a + b + c (formula lui Heron).
Rezolvare: Fie ABC un triunghi i D = prBC A . Se noteaz
A

ha = AD . Din teorema lui Pitagora generalizat se obine:


2

c = a + b 2 a DC unde a=BC, b=AC, c=AB.


a2 + b2 c2
DC =
2a
ADC dreptunghic AD 2 = AC 2 DC 2
2

a
D

b
C

)(

a2 + b2 c2
AD 2 = 1 2ab a 2 b 2 + c 2 2ab + a 2 + b 2 c 2
AD = b

2a
4 a2

2
ha =
p ( p a )( p b )( p c ) unde ha = AD .
a
a ha
Dar S =
S = p ( p a )( p b )( p c ) .
2
Aplicaia 3
S se demonstreze c n orice paralelogram ABCD suma ptratelor laturilor
este egal cu suma ptratelor diagonalelor.
Rezolvare: Fie BB1AD i AA1BC . Evident AB1 = BA1
B1 A
D
2

n ABC aplicm teorema lui Pitagora generalizat:


AC 2 = AB 2 + BC 2 2 BC BA1 (1)
n BAD aplicm teorema lui Pitagora generalizat:
BD 2 = AB 2 + AD 2 + 2 AD AB1 (2)

A1

Adunarea relaiile (1) i (2) i innd cont c AB1 = BA1 i AD = BC


AC2 + BD2 = 2AB2 + 2BC2
AC2 + BD2 = AB2 + BC 2 + DC 2 + AD2 (DC = AB, BC = AD)

- 24 -

CERCUL DE MATEMATICA

SSM
H

Aplicaia 4
S se demonstreze c G fiind centrul de greutate al ABC , M un punct
oarecare din plan se verific relaia:
MA2 + MB 2 + MC 2 = 3MG 2 + GA2 + GB 2 + GC 2 .
Rezolvare: Fie A mijlocul lui [BC].
Din relaia lui Stewart pentru MAA' i G ( AA')

MG 2 AA' = MA2 A' G + MA'2 AG AA' AG GA'


G
1
2
Dar A' G = AA' ; AG = AA'
3
3
1
2
2
B
A
MG 2 = MA2 + MA'2 AA'2
3
3
9
1
2 MB 2 + MC 2 BC 2 2
MA median n MBC MG 2 = MA2 +

AA'2
3
3
2
4
9
2
BC
2
3MG 2 = MA2 + MB 2 + MC 2
AA'2
2
3

GA median n GBC 3MG 2 = MA2 + MB 2 + MC 2 GB 2 + GC 2 2GA'2

2
AA'2
3

3
1
GA ; A' G = GA 3MG 2 = MA2 + MB 2 + MC 2 GA2 GB 2 GC 2
2
2
2
2
2
MA + MB + MC = 3MG 2 + GA2 + GB 2 + GC 2
Observaie: Folosind relaia: MA2 + MB 2 + MC 2 = 3MG 2 + GA2 + GB 2 + GC 2 se

Din AA'=

obine c MA2 + MB 2 + MC 2 este minim dac i numai dac M=G.

Probleme propuse

1. Fie ABC CU AB=34, BC=56, M ( AC ) , ( AM ) (MC ) , BM=39. S se afle


distana de la M la BC.
2. Prin vrful A al triunghiului isoscel ABC se duce o paralel la BC pe
care se consider un punct M. S se demonstreze c
MB 2 + MC 2 = 2 MA2 + MB 2 .

1 2
a + b 2 + c 2 unde G este centrul de
9
greutate al triunghiului ABC, a, b, c sunt lungimile laturilor
triunghiului iar C (O, R ) este cercul circumscris triunghiului.

3. S se demonstreze c: OG 2 = R 2

4. Aria unui patrulater convex ABCD avnd lungimile laturilor a, b, c, d


B+ D
este dat de S 2 = ( p a )( p b )( p c )( p d ) abcd cos2
(formula lui
2
Arhimede) unde 2p=a+b+c+d, S = aria lui ABCD.
Bibliografie
1. Gh. ieica Culegere de probleme de geometrie;
2. L. Nicolescu, V. Boscoff Probleme practice de geometrie

- 25 -

CERCUL DE MATEMATICA
Tem pentru grupele de performan, clasa a VIII-a
PARTEA NTREAG I PARTEA FRACIONAR A
UNUI NUMR REAL

prof. Draga Ttucu Mariana

Noiuni pregtitoare

Def. 1 Fie a R . Se numete partea ntreag a numrului a, numrul


notat [a ] , care reprezint cel mai mare numr ntreg mai mic sau egal cu a.
Deci [a ] Z , [a ] a < [a ] + 1
Se numete partea fracionar a numrului a, numrul, notat {a}, care este
egal cu diferena dintre a i partea sa ntreag [a ] .
Deci {a} = a [a]
Proprieti
Prezentm cteva proprieti care rezult din definiie.
P1. Pentru m Z , x [m, m + 1) [x] = m . P3. [x ] = x x Z , {x} = 0 x = 0 .
P2. x, y [m, m + 1) [x ] = [ y ] .

P4. [m + x] = m + [x ] {x + m} = {x}
m Z, x R .

Demonstraie
Cum [x] x < [x ] + 1 [x ] + m x + m < [x] + m + 1 x + m [[x] + m, [x ] + m + 1)
Cum [x ] + m , [x] + m + 1 ntregi consecutivi [m + x ] = m + [x ]
{x + m} = x + m [x + m] = x + m [x ] m = x [x] = {x}.

P5. [x ] + [ y ] [x + y ] [x] + [ y ] + 1 , x, y R .
Demonstraie
x = [x] + {x} ; y = [ y ] + {y}; [x + y ] = [[x ] + [ y ] + {x} + {y}] = [x] + [ y ] + [{x} + {y}]
Dar {x} + {y} [0,2) [{x} + {y}] {0,1} [x ] + [ y ] [x + y ] i [x + y ] [x ] + [ y ] + 1
Observaie: [x] : R Z

n ,
......

1,
[x ] = 0,
1,

......

n,
x + n,
.....

x + 1,
{x} = x 0,
x 1,

......

x n,

- 26 -

x [ n, n 1)
x [ 1,0 )
x [0,1)
x [1,2 )

x [n, n + 1)
x [ n,(n + 1)]
x [ 1,0 )
x [0,1)
x [1,2 )
x [n, n + 1)

CERCUL DE MATEMATICA
Aplicaii

SSM
H

13x 5 5 x + 1
Aplicaia 1. Rezolvai ecuaia:
= 3
7
5x + 1
3k 1
39k 38
Soluie: Notnd
= k, kZ x =
ecuaia devine
=k
3
5
35
39k 38
39k 38
k
< k +1 k 0
k < 1 0 39 k 38 35k < 35 4k < 73
35
35

3k 1

k {10,11,12,13,14,15,16,17,18} = A soluiile ecuaiei sunt: x =


k A
5

5 x 1 x 1
Aplicaia 2. Rezolvai ecuaia:
+ 2 [x ]
3
5x 1
5 x 1
Soluie: Fie
= m i [x ] = n ; m, n Z m
< m +1

3
3
3m + 1 3m + 4
x
,
; n x < n + 1 x [n, n + 1)
5
5
x 1
Inecuaia devine: m +
n x [2n 2m + 1, )
2
Mulimea soluiilor inecuaiei va fi reuniune de intervale de forma:
3m + 1 3m + 4
5 , 5 I [n, n + 1) I (2n 2m + 1,+ ) .

Aplicaia 3. S se arate c oricare ar fi x R are loc identitatea (Hermite):


[x] + x + 1 = [2 x]
2

Soluie: x = [x] + {x}


1
1
1
Caz 1). 0 {x} < x + = [x] + {x} + < [x] + 1 (i evident x [x] )
2
2
2
1
1

Deci x + = [x] i [2 x] = 2[x] [2 x] [x] = 2[x ] [x ] = [x] = x +


2
2

1
1

Caz 2). {x} > x + = [x] + 1


2
2

[2 x] = 2[x ] + 1 i deci: [2 x] [x ] = 2[x ] + 1 [x ] = [x] + 1 = x + 1


2

Generalizare:
[x ] + x + 1 + x + 2 + .... + x + n 1 = [nx] , x > 0 , n N *
n
n
n

x 2 + 3 x 2 1 x + 1
Aplicaia 4. S se rezolve ecuaia:
.
+
=
2
8 8
x +1
Soluie: Notm
= k Z x = 2k 1 i ecuaia devine:
2
(2k 1)2 + 3 (2k 1)2 1
k2 k 1 k2 k
+
=
k
+ +
(1)

=k
8
8
2 2
2

- 27 -

CERCUL DE MATEMATICA
1

Dar [x ] = x + = [2 x] ,
2

k2 k 1 k2 k
2
+ +
xR
= k k
2
2
2

(2)

Dar k Z i k 2 k = k 2 k

(3)

Din (1), (2) i (3) k 2 k = k k {0,2} x { 1,3}


Aplicaia 5. S se rezolve ecuaia: [x[x]] = 1 .
Soluie: [x[x]] = 1 1 x[x ] < 2
(1)
Dac x < 1 [x 2] x[x] > 2 nu verific (1).
Dac x = 1 [x] = 1 x[x] = 1 verific (1) x = 1 este soluie.
Dac 1 < x < 0 [x] = 1 x[x ] = x nu verific (1).
Dac 0 x < 1 [x ] = 0 x[x] = 1 nu verific (1).
Dac 1 x < 2 [x ] = 1 [x] = 1 x[x ] = x 1 x[x] < 2 verific (1).
Orice x [1,2 ) este soluie.
Dac x 2 [x ] 2 x[x] 4 nu verific (1)
n final, ecuaia are soluiile x [1,2) U { 1}.
Probleme propuse
1. Rezolvai ecuaiile:
x
x + 2
a)
=

x 1 x 3
1 1
1
b)
+
=
;
x [x ] {x}
x + 1 2 x 1 x 1
c)

= 6 ;
2 3
25 x 2 13 x + 4
d)
=
.
3
4

1 + na 2

1+ 5
a
2. Dac a
, s se demonstreze c: 1 +
= n,
2
a

BIBLIOGRAFIE

nN .

1. N.I.Nedi, I.M.Rdoi Algebr-exerciii i probleme, Ed.Universal, 1997;


2. C.Nstsescu, C.Ni Culegere de algebr.

- 28 -

CERCUL DE MATEMATICA

SSM
H

Tem pentru grupele de performan, clasa a VIII-a


PROBLEME DE ORDONARE
prof.Constantin Duu
prof.Ani Pavelescu
Pentru nelegerea problemelor care urmeaz este bine s reinem
urmtoarele:
1) Dac a i b sunt dou numere reale, atunci este adevrat una i
numai una din relaiile: a>b, a=b, a<b.
2) Prin max (a,b) nelegem cel mai mare dintre numerele a i b, iar prin
min (a,b) nelegem cel mai mic dintre numerele a i b:
a daca a > b
a+b+ ab

max(a, b) = a sau b daca a = b


sau max(a, b) =
2
b daca a < b

b daca a > b
a +b a b

min( a, b) = a sau b daca a = b


sau min( a, b) =
2
a daca a < b

3) Vom admite c oricare ar fi numerele reale a i b, a 1, b>0, exist un


singur numr real c astfel nct ac=b. De asemenea, vom admite c
dac a>1 atunci din ax1 >ax2 => x1>x2 i reciproc, iar dac a<1 atunci
din ax1 >ax2 => x1<x 2 i reciproc.
4) Pentru a stabili care dintre numerele reale a i b este mai mare, cnd
acest fapt nu este evident, putem folosi mai multe procedee :
a) Stabilim semnul diferenei a-b. Dac a-b>0 atunci a>b; dac a-b=0,
atunci a=b; dac a-b<0, atunci a<b.
a
b) Dac numerele sunt pozitive i b 0, comparm raportul
cu 1.
b
a
a
a
Dac
>1, atunci a>b; dac =1, atunci a=b; dac <1, atunci a<b.
b
b
b
c) n anumite cazuri, este suficient s demonstrm existena unui
numr c care s se afle ntre a i b sau b i a. De exemplu, din a<c<b
rezult a<b.

Probleme comentate:
1. Care dintre numerele : a = 11 + 2 2 + 33 + ... + 10001000 ; b = 2 2
(a i b sunt scrise n sistemul zecimal).

22

este mai mare?

16

Soluie: Avem b= 2 2 , dar 216=21026 =102464>100064=64000. Deci


b>264000.
Pe de alt parte: a= 11+22+33++10001000 < 10001000+10001000++10001000=
= 100010001000 = 10001001 < 10241001 = (210)1001=210010. Deci a<210010.
Din a<210010 i b>264000 => a<b.
Comentariu: Nu am comparat direct numerele a i b. Am artat c exist
dou numere c i d care ndeplinesc condiiile a<c, c<d, d<b, de une rezult
i ordinea a<c<d<b.

- 29 -

CERCUL DE MATEMATICA
2. S se demonstreze c (n!)2 > nn pentru orice numr natural strict mai
mare dect 2, unde prin n! se nelege n!=123n, iar n>2.
Soluie: n rezolvarea problemei vom folosi urmtoarea proprietate:
dac a b i c>d => ac>bd,
a,b,c,d>0.
2
n
Notm a=(n!) i b=n i vom avea urmtoarea scriere:
a=123(n-2) (n-1) n n (n-1) (n-2) 321 =>
a=(1n) [2(n-1)] [3(n-2)] [k(n-k+1)] [(n-2) 3] [(n-1) 2] [n1].
b=nnnn (n factori)
Scrise aa, se observ c numerele a i b au cte n factori. S comparm un
factor oarecare al primului produs cu un factor oarecare al celui de-al doilea
produs: k(n-k+1)-n=kn-k(k-1)-n=n(k-1)-k(k-1)=(k-1)(n-k)
Cum 2 k n-1 => k-1 >0 i n-k>0 deci (k-1) (n-k)>0 => k(n-k+1)>n.
Am demonstrat c a este format din factori mai mari sau cel puin egali cu
ai lui b (primul i ultimul factor ai lui a sunt egali cu orice factor ai lui b), n
rest inegalitatea este strict, deci a>b.
Comentariu: Deoarece (n!)2>nn, oricare ar fi n N\{0,1,2} => dac ecuaia
(n!)2=nn are soluii n N, atunci acestea se gsesc printre elementele mulimii
{0,1,2}. Verific numai n=1 i n=2, deci (n!)2=nn are n N soluiile 1 i 2.

n n + (n + 1)
> 1, (
3. S se demonstreze inegalitatea: n +1
n
n + (n + 1)
n +1

) n N* .

Soluie: Membrul stng al inegalitii se mai scrie :

n n
n n

(n + 1)
+ n + 1
+ n + 1
n + 1
n + 1

n n + ( n + 1) n +1 n n + (n + 1)n (n + 1)
=
=
=
n

n n
.
n n +1 + ( n + 1) n
n n n + (n + 1)n
n n
(n + 1)
n + 1
n + 1
n
+
1

n + 1

am simplificat prin (n+1)n.

n
Cum
<1 =>
n +1

<1 =>
n +1

dar n+1 <

+ n +1
n +1

n
n

n < n =>
n+1 < n+1
n +1
n +1
n

n
n
+ n +1

n +1 <
n +1
n +1

+ n +1
n + 1

>1
n
n

n +1
n + 1

n n + ( n + 1) n +1
> 1, oricare ar fi n N*.
n n +1 + ( n + 1) n

Comentariu: Inegalitatea demonstrat ne permite s comparm uor


numerele de forma a=9999+100100 i b=99100+10099.
4. S se afle max( 2
Soluie: Notm a= 2

- 30 -

, 3
3

).

, b= 3

i obinem

CERCUL DE MATEMATICA
a2=

b2= 3

(b )
2

( )

>9>8> a 2

SSM
H

=2

(a )
2

=3

( )
(b ) = (3 )
2

b2 > a2

b > a max( 2

2
2

2
3

= 2

=23=8.
=3

> 3
3

=9

, 3

)= 3

a , a 2 , unde a R.

5. S se determine max a

Soluie: Se tie c pentru a R avem |a| 0, deci numrul


Vom distinge urmtoarele situaii:

a. Dac a<0, avem a a <0 i a2>0 deci max a

c. Dac a (0,1) |a|=a a a = a a =


Pentru a (0,1) a3>a4

a , a 2 =a2.

a , a 2 =0, evident.

b. Dac a=0, avem max a

d. Dac a=1 avem max a

a este definit.

a3 > a4

a3 .

a a >a2 max a

a ,a2 = a a .

a , a 2 =1.

a , a =a2.

a 2 , daca a ( ,0] [1,+ )


2
max a a , a =
a a , daca a (0,1)

e. Dac a (1,+ ) avem a3<a4 a a <a2 max a


n concluzie avem:

Probleme propuse spre rezolvare elevilor


1. Dac a,b,c N*, pot fi fraciile

a c b
, ,
simultan subunitare?
c b a

2. S se stabileasc care dintre numerele 14523 i (23!)2 este mai mare.

Indicaie: Se folosete proprietatea c dac suma a dou numere a i b este


constant, produsul lor este maxim dac numerele sunt egale a=b.

3. Dac numerele naturale a, b, c verific relaiile :


a-2c 1 ; b-c 1; 2b-a 1, stabilii ordinea dintre ele.

Indicaie: Se nmulete a doua relaie cu 2 i se adun parte cu parte cele 3 relaii.

Bibliografie
Matematica n gimnaziu i licee,
vol. III, prof. Adrian Ghioca i Acad. Nae Teodorescu

- 31 -

CERCUL DE MATEMATICA
Tem pentru grupele de performan, clasa a IX-a
FUNCIA PARTE NTREAG,
FUNCIA PARTE FRACIONAR

prof.

D1. Mulimea nevid A R se numete majorat (mrginit superior)


dac exist un numr b R astfel nct x b, x A .
Orice numr b R cu aceast proprietate se numete majorant al lui A.
Exemple: Numerele 1,2, sunt majorani pentru mulimile
A = { 1,0,1}si B = ( ,1)
Observaie: O mulime majorat are mai muli majorani.
A1. Axioma lui Cantor.
Orice mulime nevid A R majorat admite un cel mai mic majorant
care se numete marginea superioar a mulimii A i se noteaz sup A
A = ( ,3) sup A = 3 A
Exemple:
A = ( ,3] sup A = 3 A
T1. Teorema lui Arhimede.
Pentru orice numr real x exist un numr natural n mai mare dect
x (x<n).
S presupunem prin absurd c exist un numr real x0 a.i. n x0
n N.
Aceasta ar nsemna c mulimea N este majorat i exist sup N = x0 x0 .
Prin urmare i n + 1 x 0

n N , adic n x0 1 n N ceea ce contrazice

faptul c x0 = sup N .
T2. Fiind dat un numr real x, exist un numr ntreg unic k, astfel
nct: kxk+1
D2. Acest numr ntreg k se numete partea ntreag a numrului real
x i se noteaz cu [x]. Deci
[x]x[x]+1 (*)
Demostratie: Dac x0, atunci conform proprietatii lui Arhimede exista
m N * a.i. x < m . Fie n0 cel mai mic numar din N* care satisface aceasta
condiie (x<n0) i deci n0-1 x < n0. Dac notm k=n0-1 atunci avem k x <
k+1, deci verific relaia (*).
n cazul x<0 avem 0<-x i fie n0 N * cel mai mic numr pentru care x<n0.
Dac x Z atunci n0 - 1 = -x, deci x = -n0 + 1 i k = -n0 + 1 deoarece n0 + 1 =
x < (-n0+1)+1.
Daca x Z atunci k=-n0 deoarece din n0-1<-x<n0 rezult n0<x<-n0+1
Unicitatea: S presupunem c ar exista dou numere ntregi k i k care
verific relaia (*). Fie k<k adica k-k>0 . Din inegalitile kx<k+1 rezult
k-k<1. Se ajunge astfel la absurditatea 0<k-k<1 i k-k Z.
D3. Oricare ar fi x R numrul x-[x] se numete partea zecimal a lui
x i se noteaz {x}. Din [x]x[x]+1 rezulta 0<x-[x]<1, adica 0{x}<1
D4. Funcia f : R Z dat de legea f(x)=[x], unde [x] reprezint cel mai
mare numr ntreg mai mic dect x , se numete funcia parte ntreag.
D5. Funcia f : R [0,1) dat de legea f(x)=x-[x] se numete funcia parte
ntreag.

- 32 -

CERCUL DE MATEMATICA

SSM
H

Proprieti ale funciei parte ntreag


P1. x-1<[x]x n R.
P2.[x]x<[x]+1
P3.[x+y]=[x]+y x Rsi y Z
P4.[x]+[y][x]+[y]+1

0, x Z
P5.[x]+[-x]=
1, x R \ Z
[x ] x
P6. = , daca y N *
y y

Demonstraie:
P1 i P2 rezult imediat din definiia prii ntregi a unui numr.
P3. Conform definitiei partii intregi a unui numar avem x=[x]+{x} rezulta
x+y=[x]+y+{x}. Deoarece {x} [0,1) deducem ca [x] + y x + y < [ x] + y + 1 si cum
y Z deducem ca [x+y]=[x]+y si deci P3 este demonstrata.
P4.

x = [x] + {x}, y = [ y ] + {y} [x ] + [ y ] [[x ] + [ y ] + {x} + {y}] = [x] + [ y ] + [{x} + {y}]

[x] + [ y ] + 1 [x] + [ y ] [x + y ] < [x] + [ y ] + 1

x = [x] + {x} x = [x] {x} x = [x] 1 + 1 {x}

P5. Putem scrie

[ x] = [x] 1 + [1 {x}]

[x] + [ x ] = 1 + [1 {x}] =

0 daca

1 daca

{x} = 0 adica x Z
{x} (0,1) adica x R \ Z )

x = [x] + {x}; [x] = cy + r unde 0 r y 1, deci

P6. Putem scrie x cy + r + {x}


r + {x}
=c+
y =

= c pt. ca 0 r + {x} < r + 1 y


y

y
[x ] cy + r
r
r
n
(folosim faptul ca xy-1) =
= c + = c + = c deoarece 0 < 1

y
y
y y
y
x
[x ]
Deci = c =
y
y

Proprieti ale prii fracionare a unui numr real


daca x Z
0
Pf1. Dac x R, {x} + { x} =
1 daca x R \ Z
Demonstraie: Avem x=[x]+{x}, -x=[-x]+{-x}. Adunnd egalitile obinem
0, x Z
0=[x]+[-x]+{x}+{-x}, dar cum P5 afirm [x]+[-x]=
deducem
1, x R \ Z
0, x Z
{x}+{-x}=
1, x R \ Z
Pf2. Dac x Z si y (0, ) atunci {x+y}={y}.
Demonstraie: Pornind de la y=[y]+{y} i adunnd cu x ambii membri
obinem x+y=x+[y]+{y}(1)
Dar din [y]y<[y]+1; adunnd x rezult x+[y]=x+y<x+[y]+1 rezult c n
egalitatea (1) x+{y} este partea ntreag a lui x+y. Rezult c {x+y}={y}
Pf3. Fie x Z si y R, y > 0 . Are loc {x-y}=1-{y}

- 33 -

CERCUL DE MATEMATICA
Demonstraie: y=[y]+{y} rezult
y = [ y ] {y} x y = x [ y ] {y}

(1)
x y = x [ y ] 1 + 1 {y}
Dar [ y ] y [ y ] y x [ y ] x y
(2)
[ y ] + 1 > y [ y ] 1 < y x [ y ] 1 < x y
(3)
Din (2) i (3) rezult x-[y]-1<x-yx-[y], deci n egalitatea (1) x-[y]-1 este partea
ntreag a lui x-y, de unde rezult c {x-y}=1-{y}

Probleme comentate:
1

1. Daca x R ,atunci [x] + x + = [2 x]


2

Solutie: Fie x = [x] + {x}cu{x} [0,1) .Atunci 2 x = 2[x ] + 2{x} si x +

1
1
= [x] + {x} + .
2
2

Rezulta ca avem urmatoarele cazuri:


1
1

a) 0 {x} < 0 2{x} < 1 si in acest caz [2 x] = 2[x], x + = [x ] , iar


2
2

formula este verificata


1
1

b) {x} < 1 1 2{x} < 2 atunci [2 x] = 2[x ] + 1 si x + = [x] + 1 si din nou


2
2

formula este verificata.


2. Daca x R, n N * , atunci:
[x ] + x + 1 + ... + x + n 1 = [nx].(Hermite)
n
n

l
k
k +1

Solutia(1): Fie x = [x] + {x} unde < {x} <


, k N atunci x + = [x ] pentru
n
2
2

l = 1,2,3,..., n k 1 si x + = [x] + 1 pentru l = n k ,..., n 1


n

Deci

[x] + ... + [x ]+ ([x] + 1) + ... + ([x] + 1) = n[x ] + k , iar membrul drept este egal
k =1

de ( n k ) ori

de. k .ori

cu [nx] = [n[x ] + n{x}] = n[x ] + k si astfel formula este verificata.


1
n 1

Solutia(2): Consideram f : R R; f (n) = [x]+ | x + + ... + x +


[nx]. Se
x
x

1
1
1

verifica faptul ca f (n) = 0, n 0, si. f x + = f (n) . Deci f are perioada .


x
n
n

1
k
k +1
k
Pentru ( )x R , ()k Za.i. x + = k x <
x = + , [0,1) ,
1
n
n
n

n
1

deci f (n) = f k + = f ( ) = 0 , deci f (n) = o, n R de unde se obtine


n

identitatea lui Hermite

- 34 -

CERCUL DE MATEMATICA
n + 2n
n + 1 n + 2
3. Sa se arate ca pentru orice n natural,
+
+
...
+
n+1 = n .
2
2 2
2

SSM
H

n + 2k n
1 n n
Solutie: k +1 = k +1 + = k k +1 , k {1,2,..., n}. Adunand egalitatile
2 2 2
2
2
n + 2n n n
n + 1 n + 2
obtinute, avem:
+
+
...
+
n+1 = 0 n+1 , dar
2
2 2
2
2 2

n
< 1, n N n +1 = 0 si astfel obtinem egalitatea
2
2
n
n + 2
n + 1
+
...
+
n+1 = [n] = n
2

4. Demonstrati ca pentru orice numar real x are loc relatia:


x + 3 x + 4 x + 5 x + 1 x + 1
6 6 + 6 = 2 3
C.Martici,Targoviste
x +1
1
1
2

Solutie: Notm
= y, avem : y + y + + y + = [3 y ] [2 y ] ceea ce se
6
3
2
3

1
1
2

deduce din identitatea lui Hermite: [2 y ] = [ y ] + y + si[3 y ] = [ y ] + y + + y +


2
3
3

5. Daca [x + y ] = [x ] + [ y ]si[ x y ] = [ x] + [ y ] atunci xZ sau yZ.


Solutie: Presupunem ca
(1)
x Zsi. y Z [x ] + [ x] = 1, [ y ] + [ y ] = 1 [x ] + [ x ] + [ y ] + [ y ] = 2
0, x + y Z
Dar [x ] + [ x ] + [ y ] + [ y ] = [x + y ] + [ x y ] =
(2)
1, x + y Z
Din (1) i (2) deducem ca -2=0 sau -2=-1 (fals)
Deci presupunerea fcut este fals i astfel se obine concluzia.
6. Daca x si y sunt numere reale pozitive, atunci [xy] [x] [ y ]
Solutie: x = [x] + ; y = [ y ] + ; , [0,1) xy = [x ] [ y ] + [x] + [ y ] +
deci [ny ] = [x] [ y ] + [ [x] + [ y ] + ] . Cum
x, y 0 si , [0,1) [ [x] + [ y ] + ] 0 [xy ] [x ] [ y ]
7. Daca x si y sunt numere reale atunci [x y ] [x ] [ y ] [x y ] + 1
x = [x] + {x}; {x} = [0,1)
Solutie:
y = [ y ] + {y}; {y} = [0,1)
Si deci x y = [x] [ y ] + [x y ] = [x ] [ y ] + [ ]
Deoarece , [0,1) ( 1,1) [ ] { 1,0}(1)
Inegalitatea [x y ] [x] [ y ] [x ] [ y ] + [ ] [x ] [ y ] [ ] 0 ceea ce este
adevarat conform (1)
[x] [ y ] [x y ] + 1 0 1 + [ ]ceea ce este adevarat conform(1)
0

n +1

8. Pentru care n este adevarata egalitatea:

[ n ] =

n?

- 35 -

CERCUL DE MATEMATICA
Solutie: Fie a N pentru care a n < n < (a + 1) ,iar de aici
n

n < a +1

n = a .Pe de alta parte

2
a 2 n < (a + 1) a
n < a +1
n =a

Deci relatia are loc pentru orice n 1


9. Sa se rezolve ecuatia: x + [x ] = [ x [x]]
a 2 < n < (a + 1) a <
2

[ ]

[ ]

[ ]

Solutie: Cum x x > [x ] deducem x [x ] = x [x ]

Pe de alta parteb [ x [x ]] = [ x ] [x ],deci ecuatia devine x = [ x ] cu solutie


pentru orice x Z
10. S se rezolve n numere naturale prime:
Solutie: Facem observatia ca:

[ 1]+ [ 2 ]+ ... + [

[ x ] = [ x + 1] = ... =
2

(x + 1)

x2 1 = y

1 = x; x N ,

iar membrul stang al ecuatiei il scriem sub forma


1 + 2 + (1 + 1)2 1 + 4 + 5 + ... + (2 + 2 )2 1 + ... +

x 1
( x 1)2 + ( x 1)2 + 1 + ... + x 2 1 = k (2k + 1) = ( x 1) x(4 x + 1)

k =1
6
Ecuatia devine ( x 1)x(4 x + 1) = 6 y .De aici x=2 sau x=3 sau x=y.Pentru
x=2,y=3,iar daca x=3,y=13.Cazul x=y este imposibil
11. Sa se rezolve ecuatia: x 5 + [x ] 5 = 0

[ ] [ ]

[ ] [ ]

Solutie: Pentru x 2 se obtine x 5 x [x ] 16[x] [x] + 15[x] [x] + 30 [x ] + 5


Daca x 1 se obtine x 5 < x 4 ([x] + 1) [x ] + 1 < [x ] + 5 . Deci solutiile posibile ale
4

ecuatiei se afla in intervalul (1,2 ) , iar ecuatia devine x 5 6 = 0 , adica x = 5 6 .

12. Sa se arate ca log 2 4 78 + 4 78 + ... + 4 78 + 4 78 = 1, (n radicali), oricare ar fi

*
n N ,unde [x] reprezinta cel mai mare numar intreg mai mic decat x.

Solutie: Avem 2 < 4 78 < 3 , iar de aici , 2 < 4 78 + 4 78 < 4 78 + 3 < 4 81 < 3
Deci 2 < 4 78 + 4 78 + ... + 4 78 + 4 78 < 3. Afirmatia se verifica prin inductie.Deci

4
4
4
4
4
4
78 + 78 + ... + 78 + 78 = 2 log 2 78 + 78 + ... + 78 + 78 = 1

{(

13. Sa se calculeze lim 3 + 2 2


n

) }, ({a}reprezinta partea fractionara a lui a).


n

Solutie: Daca dezvoltam dupa binomul lui Newton, obtinem

(3 + 2 2 ) = A + B 2 ; (3 2 2 )
{(3 + 2 2 ) }= {A + B 2}= {B 2}
n

= A B 2 , A, B N *

{(3 2 2 ) }= {A B 2}= 1 {B 2}
De unde prin adunare obtinem
{(3 + 2 2 ) }+ {(3 2 2 )}= 1
0 < 3 2 2 < 1 0 < (3 2 2 ) < 1 (3 2 2 ) = 0 (3 2 2 ) = {(3 2 2 ) }

- 36 -

(1)

CERCUL DE MATEMATICA

Dar 3 2 2

{(

lim 3 + 2 2
n

{(

0 32 2

) }= 1

) } 0 . i innd cont de relaia (1) deducem c


n

SSM
H

Probleme propuse

1
1
1. Sa se arate ca 1 +
+ ... +
= 1998
2
1000000

2. Sa se gaseasca partea intreaga a numarului a = 23 2 + 3 4


3. Sa se gaseasca partea intreaga a numarului b = log 2 3 + log 3 5 + log 5 8

] [

n +1
2
5. Fiind dat m un numar natural, sa se arate ca functia f : N R definita

4. Sa se demonstreze inegalitatea: lg 1 2 + lg 2 3 + ... +

n(n + 1) < n lg

prin f ( n) = m n 2 este injectiva.


6. Sa se arate ca oricare ar fi numerele reale x1,x2,...xn, avem inegalitatile
[xi ]2 + 3 > 2 xi , i = 1,2,..., n [x1 ]2 + [x2 ]2 + ... + [x n ]2 + 3n > 2(x1 + x 2 + ... + xn )

m 2m
(n 1)m
*
7. Se cere suma + + ... +
; n, m N i sunt prime ntre ele.
n
n
n

x a x
x
8. Dac 0 < a < b, c > 1 s se rezolve ecuaia
= lg a
c
b
b

x
9. S se rezolve ecuaia x [x] + = a, a R.
n
10. S se rezolve ecuaia tg [n ] tg {n} = 1

a + x a
11. Fie a, b N . S se determine mulimile A = x Z ,
=
b b

b 1

a + k
a + x a
B = x Z ,
=
+
1
,
apoi
s
se
stabileasc
identitatea

= a
b b
k =0 b

12. Fie n N * , f : R R, f ( x) = x n [x ], x R. S se determine f(R)


L.Panaitopol
1
1 1
13. S se rezolve ecuaia:
= +
{x} x [x ]
M.Andronache si M.Chirita
a2 b2 c2
14. Aflai numerele a, b, c N * cu proprietatea + + = 0
2b 2c 2a

[ ]

x2 y2 z2

2
15. Aflai x, y, z N * pentru care
+
+
=
.
z + y x + z x + y x + y + z
Bibliografie

1. Dumitru Busneag, Ioan Maftei - Teme pentru cercurile si concursurile de


matematica ale elevilor
2. Mircea Ganga - Teme si probleme de matematica.
3. Gazeta Matematica-seriaB 2000-2002

- 37 -

CERCUL DE MATEMATICA
Tem pentru grupele de performan, clasa a X-a
Ecuaii exponeniale i logaritmice
Prof. Nedeianu Dan
n cele ce urmeaz ne propunem s prezentm diverse metode i
tehnici de abordare a unor ecuaii exponeniale sau logaritmice, ntlnite la
olimpiade i concursuri colare. Pe lng metodele clasice, ce in de
monotonia funciilor (n sensul de a arta c o ecuaie exponenial sau
logaritmic are soluie unic), se pot pune n eviden i alte tipuri de
metode interesante.
Metoda I (folosind inegaliti)
este o metod care const n aplicarea unor inegaliti cunoscute, i de cele
mai multe ori, cazul de egalitate (pentru care se realizeaz inegalitatea), ne
ofer chiar soluia ecuaiei;
4
3x + 1
Exemplul I.1: S se rezolve ecuaia: 8 + log 2 2
=x+ 4
x2
x + 11
(Prof. Nedeianu Dan, Concursul Gh. ieica -faza judeean 2001)
4
3x + 1
Soluie: Ecuaia se mai scrie: 6 + log 2 2
=x+2+ 4
x2
x + 11
4
Cu inegalitatea mediilor, avnd x+2+ 4
5 (cu egalitate cnd x=3)
x2
3x + 1 1
3x + 1
Apoi avem (x-3)2 0 2
6 + log 2 2
5 (cu egalitate cnd x=3)
x + 11 2
x + 11
4
3x + 1
Deci x+2+ 4
5 6 + log 2 2
, deci x=3 este soluie unic.
x2
x + 11
x
Exemplul I.2: S se rezolve ecuaia: x log 16 + 8x-4=0
(Concurs Vrnceanu-Procopiu Bacu 2001)
2
Soluie: Din notaia log16x=y 24y + 24y+3=4
2

Cu inegalitatea mediilor, vom avea: 24y + 24y+3 2


1
1
4 y 2 + 4 y +3
4 2 2
(2y+1)2 0 y = x =
2
4

24 y

+ 4 y +3

i atunci

Metoda a II-a (folosind substituii)


este o metod care const n efectuarea unor substituii, care au ca scop
obinerea de ecuaii (n noua variabil) clasice.
Exemplul II.1: S se rezolve ecuaia: log 14 x + 3 x + 6 x = log 64 x
(O.L.M. Sibiu 2002)
6t
t
t
Soluie: Notnd log 64 x = t
x=2
log14 (8 +4 +2t)=t , adic
8t+4t+2t=14t , cu t=1 soluie unic, deci x=64.
3 + 2 3x 1
= log 3 (3+2x-3x2)
Exemplul II.2: S se rezolve ecuaia: 1+3 log 2

(prof. Nedeianu Dan, revista Cardinal 2003)

- 38 -

CERCUL DE MATEMATICA
Soluie: Notm 3 x 1 =t 0 -3x2+2x+3=

10 t 2
i ecuaia devine:
3

SSM
H

; se impune 10-t2>0 t [0, 10 )

10 t 2
2t + 3
ns pe [0, 10 ) funciile f(t)= 1+3 log 2
i
g(t)=
log

3
5 sunt strict
5

cresctoare, respectiv strict descresctoare, ceea ce implic unicitatea


2
soluiei t=1 pentru ecuaia (*) x1=0; x2=
3
2t + 3
(*) 1+3 log 2
= log 3
5

10 t 2

Metoda a III-a (folosind descompuneri)


este o metod care const n unele artificii de calcul, prin care ecuaia
iniial f (x) =0 se aduce la forma g ( x)h ( x) =0, unde g ( x), h( x) sunt funcii
exponeniale/logaritmice mai elementare dect funcia iniial f (x) .
Exemplul III.1: S se rezolve ecuaia: 9x + (x-3)3x + 2 = 4x + (x-1)2x + x
(Revista Arhimede, 2002)
Solutie: Notnd 3x = a i 2x = b a2 + (x-3)a + 2 = b2 + (x-1)b + x
Completnd la un ptrat expresiile a2 + (x-3)a , respectiv b2 + (x-1)b se va
2

x 3
x 1

obine: a +
= b +
(a-b-1)(a+b+x-2)=0
2
2

(3x-2x-1)(3x+2x+x-2)=0, de unde x {0,1}


Exemplul III.2: S se rezolve ecuaia:

3x + 1 2 x + 1
x 1 = 1
2 x 1
3
(Concurs Gh. Mihoc - 1998)

2(b + 1) 3( a + 1)

= 1 ; dup grupri i unele calcule


a
b
(considernd eventual ecuaia n variabila a, cu parametrul b) se va obine
(2b-3a)(a+b+1)=0 (2 3 x 3 2 x )(2 x + 3 x + 1) = 0 i atunci x=1 soluie unic.

Soluie: 2x=a, 3y=b

Metoda a IV-a (pentru ecuaii reductibile la forma f ( x) = f ( y ) sau


f ( x) = f ( g ( x)) )
este o metod mai nou, care const n prelucrarea ecuaiei incercnd s o
aducem la o form de genul amintit, unde funcia f este strict monoton
sau injectiv.
2
x 2 3x
Exemplul IV.1: S se rezolve ecuaia: 2 x 3 x +
=1
2x
(O.L.M. Bucureti 2001)
2
x2 2 x
2
x
Soluie: ecuaia se mai scrie: 2
+ x 3 x = 2 x + 2 x = ( x 2 2 x) + 2 x 2 x
Notnd f (x) =x+2x (care este strict cresctoare),avem f (x) = f ( x 2 2 x) ,
deci x2-2x=x x {0,3}
Exemplul IV.2: S se rezolve ecuaia: log a (b x + a b ) = log b (a x + b a ), a>b>1.
(O.L.M Tulcea, 2000)

- 39 -

CERCUL DE MATEMATICA
Soluie: Notnd cu y ambii logaritmi, implic a y = b x + a b i b y = a x + b a ,
de unde a x + b x = a y +b y . Considernd f (t ) = a t + b t (strict cresctoare), se
deduce x=y a x = b x + a b x = 1 soluie unic.
Metoda a V-a: (pentru ecuaii reductibile la forma f (x) = f 1 ( x) )
este o metod inedit, ce presupune aducerea ecuaiei la o form
f (x) = f 1 ( x) , unde f bijectiv, caz n care vom avea practic de rezolvat
ecuaia f (x) =x
Exemplul V.1: S se rezolve ecuaia: 2

lg x

+ 8 = ( x 8) lg 2

Soluie: Considernd f : (8, ) (8, ), f ( x) = 2 lg x + 8, x > 8 , se arat simplu c


f este strict cresctoare, bijectiv, inversabil cu inversa
1

f 1 : (8, ) (8, ), f 1 ( y ) = ( y 8) lg 2
Ecuaia devine: f ( x) = f 1 ( x) i innd cont c graficele funciilor f i f 1
sunt simetrice fa de dreapta y = x
f ( x) = x 2 lg x + 8 = x (lg x = t ) : 2 t + 8 = 10 t t = 1 i x=10 soluie unic
Pentru aprofundarea acestui material propunem cititorilor un set de
probleme care se rezolv folosind raionamentele anterioare.
S se rezolve ecuaiile:
1.

1 4x 2 + 4 x =

2. 2000 x

5 x + 2

1
2

(OLM Mehedini 2003)

+ 2000 x

+3 x + 2

1
4

125

(OJM Mehedini 2000)

3. x log 2 3 log 2x = 2 x

(OJM Braov 2003)

4. log 8 1 + x = log

(OLM Teleorman 2002)

x
27

5. 4 x + 15 x = 10 x + 9 x
2x
1
1
6.
= x
+ x
x
x +1
x
12 2 3
4 +1 6 1
4
2
x
7. 3 x 5 x +3 = 2
x +2
2
log32
8. ( x + 2) ( x + 3) log3 = 1

(OLM Harghita 2003)


(OLM Sibiu 2000)
(Concurs Al. Cojocaru 2003)
(OLM Brila 2001)

Bibliografie:
1) D. Brnzei, V. Gorgot
Matematica n concursurile colare (colecie 1998-2003)
2) M. Ganga
Manual clasa a X-a

- 40 -

CERCUL DE MATEMATICA

SSM
H

Tem pentru grupele de performan, clasa a XI-a


ASUPRA DETERMINANTILOR MATRICELOR FORMATE
NUMAI CU ELEMENTELE MULTIMII {-1 , 1}
Prof. Gheorghe Cainiceanu
Vom nota in cele ce urmeaza An{-1 , 1} multimea matricelor de ordin n
ce contin doar elementele 1 si 1 si cu Un = {det A | AAn{-1 ,1}}.
2
PROPOZITIA 1. Numarul elementelor multimii An{-1 ,1} este 2 n .
Demonstratie. O matrice este o functie
f : {1,2,n}2 {-1 ,1}
2
deci multimea An{-1 ,1} are 2 n elemente.
PROPOZITIA 2. a) Multimea Un contine pe 0.
b) Daca x este element in Un atunci si x este in aceasta multime.
Demonstratie. a) O matrice avand doua linii formate numai cu 1 are
determinantul 0.
b) Daca det A =x prin schimbarea a doua linii intre ele ,noua matrice A va
avea det A=-x.
OBSERVATIE. Daca an este cel mai mare element din Un , avem ca
numarul de matrice cu determinantul an este egal cu numarul de matrice cu
determinantul an.
PROPOZITIA 3. Daca A este o matrice din An{-1 ,1} atunci 2n-1 este un
divizor pentru det A.
Demonstratie. Adunand prima linie la toate celelalte linii obtin numai 0 si 2.
In general pot scoate factor comun 2 pe fiecare linie si rezulta asertiunea
dorita.
PROPOZITIA 4. Avem ca U3 = {-4 ,0 ,4}.
Demonstratie. Pentru orice x U3 din P.3 deducem ca 4 divide pe x.
Daca A este o matrice din A3{-1 ,1} deducem ca:
det A = () a1(1) a2(2) a3(3)
S 3

deci o suma de 6 termeni ce pot fi 1 sau 1 .Deci avand in vedere ca det A se


divide cu 4 poate fi cel mult 4 si deci det A este 4 ,0 ,sau 4.Daca voi gasi o
matrice cu determinantul de valoare 4 propozitia va fi demonstrata..
1 1 1

Consider A= 1 1 1 si observam ca det A=4.


1 1 1

TEOREMA 5. Pentru orice k numar intreg cuprins in intervalul [2-n ,


n-2] avem ca :
2n-1 k Un.
Demonstratie. Consideram matricea care are pe diagonala principala 1 si-n
rest numai 1. Calculam determinantul si obtinem (-1)n 2n-1 (n-2) si deci
aceasta valoare este in Un. Deducem ca si opusa acestei valori este in Un.
Schimband n in n-1 deducem ca 2n-2 (n-3) Un-1 , deci exista o matrice
B=(bij) in multimea An-1{-1 ,1} astfel incat det B = 2n-2 (n-3).

- 41 -

CERCUL DE MATEMATICA
Vom considera acum matricea
a2
a 3 ....
an
a1

b11
b12 .... b1n 1
b11
A= b21
b21
b22 .... b2 n1

...
... ....
...
...
b

n11 bn11 bn12 .... bn1n 1


cu a1 , a2 , an din multimea {-1 ,1}. Dezvoltam determinantul lui A dupa
prima linie si obtinem:
det A = a1 det B a2 det B = (a1-a2) det B.
Iau acum a1=1 , a2 = -1 si voi obtine ca det A= 2n-1 (n-3) si cu acest procedeu
din aproape in aproape putem construi matricile dorite.
APLICATII
1) Fie m si n numere naturale nenule. Sa se determine numarul
matricelor de m linii si n coloane formate numai cu elementele 1 si 1
cu proprietatea ca produsul tuturor elementelor de pe fiecare linie
precum si produsul tuturor elementelor de pe fiecare coloana este 1.
2) Fie A o matrice patratica de ordinul 3 ,ale carei elemente sunt 1 si 1.
Sa se arate ca det A este numar par.Sa se determine vloarea maxima
pe care o poate lua det A.
3) Fie A o matrice patratica de ordin n ale carei elemente sunt numai 1 si
1. Sa se arate ca det A este numar intreg multiplu de 2n-1.
4) Sa se calculeze valoarea maxima respectiv minima a determinantilor
de ordin 4 ale caror elemente sunt numai 1 si 1.
Bibliografie
[1] C.Nastasescu, M. Brandiburu, C. Nita, D. Joita
Culegere de probleme pentru liceu, Ed. Rotech Pro 1996
[2] Gazeta Matematica Metodica 1988

- 42 -

CERCUL DE MATEMATICA
Tem pentru grupele de performan, clasa a XII-a
ASUPRA UNOR FUNCTII CU PROPRIETATEA LUI
DARBOUX

SSM
H

prof.Ticui Ovidiu

Un rezultat interesant i cu foarte multe aplicaii n studiul funciilor


cu proprietatea lui Darboux este urmtorul:
Teorem(W. Jarnik) Fie I R un interval, iar f , g : I R dou funcii
f
care admit primitive pe I astfel nct g ( x) 0, x I .Atunci funcia
are
g
proprietatea lui Darboux pe intervalul I.
Demonstraie:
Avnd n vedere c funcia g admite primitive pe intervalul I, funcia g are
proprietatea lui Darboux pe intervalul I, iar pe de alt parte g ( x) 0, x I ,
deci g pstreaz semn constant pe intervalul I.
Presupunem de exemplu c g ( x) > 0, x I .
f
f

f
f
Fie a, b I , a < b, (a ), (b) (b), (a ) f (a ) g (a ) < 0 < f (b) g (b)
g
g
g
g

sau f (b) g (b) < 0 < f (a ) g (a).


(1)
Cum f, g admit primitive pe intervalul I f g admite primitive pe
intervalul I f g are proprietatea lui Darboux pe intervalul I .
(2)
Din (1) i (2) c (a, b ) I astfel nct f (c) g (c) = 0 c I astfel nct
f
f
(c ) =
are proprietatea lui Darboux pe intervalul I.
g
g

1
x2 +1
sin cos x, x > 0
Aplicaia 1. Fie f:R R, f ( x) = x
, g:R R , g ( x) =
. Artai
ex
0, x 0

1
e x sin cos x
f
f

x
: R R, ( x) =
c funcia
, x > 0 are proprietatea Darboux pe R.
2
g
g
x
+
1

0, x 0
Demonstraie:
Vom arta c funcia f admite primitive pe R.Avem c

1
1
1
1
1
2
2
2
sin x cos xdx = x cos x cos xdx = x cos x cos x 2 x cos x cos xdx + x cos x sin xdx
1
1

2
x cos cos x, x > 0
x cos sin x, x > 0
Fie f1 , f 2 : R R, f1 ( x) =
, f 2 ( x) =
x
x
0, x 0
0, x 0
0 f1 ( x) x 0 f 1 (0 + 0) = f1 (0 0) = f 1 (0) = 0 f 1 continu n x=0.
x0
x >0

- 43 -

CERCUL DE MATEMATICA
Funcia f1 este continu pe R i fiind continu i n punctul x=0 f 1 este
continu pe R f 1 admite primitive pe R i fie F1 o primitiv a sa pe R.
Analog se arat c funcia f 2 admite primitive pe R i fie F2 o primitiv a sa
pe R. Dac funcia F este o primitiv a funciei f pe R, atunci avem c :
1
2
x cos cos x 2 F1 ( x) + F2 ( x) + C , x > 0
f admite primitive pe R (1)
F ( x) =
x
2 F1 (0) + F2 (0) + C , x 0
Evident g este continu pe R g admite primitive pe R i g(x)>0, x R (2)
f
Din (1) i (2), (Teorema W.Jarnik)
are proprietatea lui Darboux pe R.
g
1

x2 + x +1
ln x sin , x 0
Aplicaia 2. Fie f : R R, f ( x) =
i g : R R, g ( x) =
.
x
e sin x
0, x = 0
1
sin x
e ln x sin

f
f

x
Artai c
: R R, ( x) =
, x 0 are proprietatea Darboux pe R.
2
x
+
x
+
1
g
g

0, x = 0
Demonstraie:

1
1
1
1
1

2
2
ln x sin x dx = x ln x cos x dx = x ln x cos x 2 x ln x cos x dx x cos x dx
1
1

x ln x cos , x 0
x cos , x 0
Fie f1 , f 2 : R R, f1 ( x) =
.Evident cele
, f 2 ( x) =
x
x
0, x = 0
0, x = 0

dou funcii sunt continue pe R i notm cu F1 , F2 primitivele funciilor


respective.Dac F este o primitiv a funciei f, atunci avem c:
1
2
x ln x cos 2 F1 ( x) F2 ( x) + C , x 0
F ( x) =
f admite primitive pe R (1)
x
2 F1 (0) F2 (0) + C , x = 0
Funcia g este continu pe R, deci admite primitive pe R i g(x)>0, x R (2)
f
Din (1) i (2), conform teoremei lui W.Jarnik
admite primitive pe R.
g
y

1
( x)dx = M i funciile
y y
0

Aplicaia 3. Fie funcia continu : R R a.. () lim

1
x2e x + 1
,x 0
, x 0

f: R R, f ( x) = x
, g : R R, g ( x) = chx
.
M , x = 0
1, x = 0

- 44 -

CERCUL DE MATEMATICA

1
chx

f
f

x
Artai c
: R R, ( x) = 2 x , x 0 are proprietatea Darboux pe R.
g
g
x e +1
M , x = 0

SSM
H

e x + e x
, funcia cosinus hiperbolic)
2
Demonstraie:

1
1
1
2 1
Fie o primitiv funciei dx = x dx = x 2 + 2 x dx .
x
x
x
x
1
x , x 0
Considerm funcia h:R R, h( x) = x
.
M , x = 0

(ch:R R, ch( x) =

1
1
1
Avem c h(0 + 0) = lim x = lim ( y ) = lim ( s )ds = M = h(0).
x0
y

y
y0
x
x >0
Analog se arat c h(0-0)=h(0) h este continu n punctul x=0 i cum h
este continu i pe R h este continu pe R. Fie H o primitiv a funciei h.
Dac F este o primitiv a funciei f, atunci
2 1
x + 2 H ( x ) + C , x 0
(1)
F ( x) =
f admite primitive pe R.
x
2 H (0) + C , x = 0

Funcia g continu pe R g admite primitive pe R i g(x)>0, x R (2)


f
Din (1) i (2), i teorema W.Jarnik
are proprietatea lui Darboux pe R.
g
Probleme propuse:

x
3 1
e cos x
, x 0 are prop. Darboux pe R.
1. Artai c h:R R, h( x) = 2 2
ln ( x + x + 3)

0, x = 0

31
ln 2 ( x 2 + x + 3)
cos , x 0
(indicaie: fie funciile f , g :R R, f ( x) =
,
i
g
(
x
)
=
x
x
e
0, x = 0
se arat c ndeplinesc condiiile din enunul teoremei lui W.Jarnik)
4
1
1
( x + 1 + x 2 + 2003) x

,x 0
2. Artai c h :R R, h( x) =
, unde :R R
sh
(
x
)

0, x = 0

( x)
( x)
= lim
= 0 , are proprietatea lui
x

x
x
e x ex
Darboux pe R. (sh:R R, sh ( x) =
funcia sinus hiperbolic).
2
este derivabil cu proprietatea lim

- 45 -

CERCUL DE MATEMATICA
1
sh ( x)
, x 0
(indicaie: f,g:R R f ( x) = x
i g ( x) =
, se arat c
1
4
0, x = 0
x +1+ 2

x + 2003
ndeplinesc condiiile din enunul teoremei lui W.Jarnik).
1
1
4
x 2 +1
2
1 ( x + 1)a
x
3. S se arate c funcia h:R R, h( x) = e sin x ln(e x + 2) , x 0 , a > 0, a 1

0, x = 0
are proprietatea lui Darboux pe R.
12
1
ln(e x + 2)
e x sin , x 0
(indicaie: f,g:R R f ( x) =
i g ( x) =
, a > 0, a 1 , se
x
1
0, x = 0
4
x 2 +1
( x + 1)a

arat c ndeplinesc condiiile din enunul teoremei lui W.Jarnik).

Bibliografie:

1. Mircea Ganga : Teme i probleme de matematic, Editura Tehnic, Bucureti, 1991.


2. Mihail Megan : Bazele analizei matematice, vol.2, Editura Eurobit, Timioara, 1997.

- 46 -

CERCUL DE MATEMATICA

SSM
H

Tem pentru grupele de performan, clasa a XII-a


Despre polinoamele ciclotomice
prof. Daniel Sitaru
Definiia 1. Fie nN*. Dac a,b Z, spunem c a este congruent cu b
modulo n i scriem: ab (mod n) dac a-b se divide cu n.
Numrul n se numete modulul congruenei. Relaia de congruen modulo
n este o relaie binar pe Z, reflexiv, simetric i tranzitiv.
Definiia 2. Fie nN*. Mulimea de numere ntregi {a0, a1, ,an-1}
se numete sistem complet de resturi modulo n dac aiaj (mod n) pentru
orice ij. Mulimea de numere naturale n={0, 1, 2,..,n-1} este un
sistem complet de resturi modulo n.
Teorema 1. Fie {a0, a1, ..,an-1} un sistem complet de resturi modulo n.
Dac a,bZ i (a,n)=1, atunci {aa0+b, aa1+b, ..,aan-1+b} este un sistem
complet de resturi modulo n.
Demonstraie: Se poate demonstra uor c:
a)
ab (mod n) i cd (mod n) acbd (mod n) i acbd (mod n)
b)
acbc (mod n) i (c,n)=1 ab (mod n)
c)
ab (mod n) apbp pentru orice pN*.
Fixm ij. Din aiaj (mod n) rezult aai + b aaj + b (mod n) de unde
na(ai-aj). Dar (a,n)=1 nai-aj aiaj (mod n). Contradicie.
Teorema este demonstrat.
Definiia 3. Fie nN*. Definim (n)indicatorul lui Euler al numrului
n astfel: (n)=card{rrn ; (r,n)=1}. Sunt cunoscute urmtoarele rezultate:
1
Teorema 2. Dac p numr prim i eN* atunci (pe)=pe(1- ).
p
Teorema 3. Indicatorul lui Euler este funcie multiplicativ:
- (mn)=(m) (n) pentru orice m, n N* nct (m,n)=1
- (n1,n2,.,np)= (n1) (n2). (np) pentru orice n1, n2,
, npN* nct (ni, nj)=1 pentru orice ij.
Teorema 4. Dac mN* i p1, p2,,pn divizorii primi distinci ai lui m

1
1
1
1
atunci:
(m)=m(1- )(1- )..(1)=m 1 .
p1
p2
pn
p
pm

Teorema 5. (Euler) Dac nN* i aZ nct (a,n)=1 atunci a (n)1 (mod n).
Teorema 6. (Fermat) Dac pN* este un numr prim i aZ nct p nu
divide a atunci: ap-11 (mod p).
Teorema 7. (Wilson) Dac p este un numr prim, numrul p divide (p-1)!+1.
Teorema 8. Fie Un={zzC ; zn=1; n2}. n aceste condiii (Un, ) este grup
abelian (numit grupul rdcinilor de ordinul n ale unitii).
n
Demonstraie: Fie z1, z2 Un ( z1 z 2 ) = z1n z 2n = 1 1 = 1 , de unde z1z2Un.
1
Elementul neutru este 1. Simetricul elementului zUn este .
z
n

1 1
1
= n = = 1.
1
z
z

Din zn=1 z0.

- 47 -

CERCUL DE MATEMATICA
1
3
Exemple: U2={1, -1}; U3={1, , 2}; = + i
; U4={1, I, -1, -i}.
2
2
Teorema 9. Mulimea Un admite i exprimrile:
Un={1, 1, 2,.,n--1}= {1, , 2,.,n-1} unde
2 k
2 k
2
2
k = cos
+ i sin
; k {0,1,2..........., n 1} iar = cos
+ i sin
n
n
n
n
este rdcina primitiv de ordinul n a unitii.
Demonstraie:
2

2
2
4
4
2
2
2
1 = cos
+ i sin
= ; 2 = cos
+ i sin
= cos
+ i sin
=
n
n
n
n
n
n
3
4
n 1
n
3 = ; 4 = ;...........; n1 = ; = 1.

Teorema 10. n grupul multiplicativ al numerelor complexe nenule (C*,)


ecuaia zn=1 are exact n soluii distincte. n aceste condiii:
a. Un este subgrup n (C*,)
b. Orice subgrup finit cu n elemente al lui (C*,) este egal cu Un
c. Un Um dac i numai dac n divide m
d. UnUm=Ud, unde d=(m,n).
Demonstraie:
n

1 1
1
= n = =1
z
1
z
b. Dac HUn este un subgrup finit cu n elemente atunci pentru orice zUn.
Deoarece card H=card Un avem H=Un.
c. Dac Un Um deducem c Un este subgrup al lui Um. Din teorema
Lagrange pentru grupuri finite deducem c n divide m. Reciproc, dac n
divide m, atunci m=kn cu k natural. Fie zUn z n=1. n aceste condiii
zm=(zn)k=1 deci zUm i cum zUn , deducem c Un Um.
d. Din faptul c U n i Um sunt subgrupuri finite atunci UnUm este subgrup
finit n (C*,). Din b. exist dN nct UnUm este subgrup finit n (C*,).
Artm c d=(m,n).
Din Ud Un conform c. avem d divide n i d divide m. Rezult c d este un
divizor comun pentru m i n.
Fie d un divizor comun arbitrar pentru m i n.
Din c. avem: Ud Un i Ud Um.
Rezult: Ud UnUm=Ud de unde d divide d adic d=(m,n).
Definiia 4. Fie nN*. Notm cu m1, m2,,mr numerele naturale mai mici
dect n i prime cu n. Cele r=(n) rdcini ale unitii de ordinul n sunt de
2m j
2m j
forma: z m j = cos
; j = 1, 2, , r i se numesc rdcini
+ i sin
n
n
primitive de ordinul n ale unitii. Fiecare din aceste r rdcini este un
generator al grupului ciclic (Un,).
r
2m j
2m j

se numete cel deDefiniia 5. Polinomul n ( z ) = z cos


i sin
n
n
j =1
a. Dac z1, z2 Un ; (z1z 2)n=

z1n z 2n = 11=1, z Un

al n-lea polinom ciclotomic i are gradul r=(n).

- 48 -

CERCUL DE MATEMATICA
Prin Pn se nelege mulimea rdcinilor primitive de ordinul n ale
unitii: PnUn i n ( x ) = ( z u ) .
uPn

Teorema 11. Dac nN* atunci z 1 =


1

SSM
H

(z ).
n

d n

Demonstraie: Artm c familia de mulimi {Pdd divide n}N este o


partiie a mulimii Un.
1. U n = U Pd .
d n

Dac d0n atunci pentru orice Pd 0 avem Un ( d 0 = 1 implic n=1), ceea


ce nseamn c Pd 0 Un. Rezult

UP

Un .

d n

Pentru cealalt incluziune considerm U n ;


2 k
2k
= cos
+ i sin
, cu 0kn-1. Simplificnd fracia k/n prin (k,n)
n
n
obinem o fracie ireductibil q/d0 unde (q, d0)=1 i d0n. Atunci
2 q
2q
= cos
+ i sin
Pd 0 U Pd .
d0
d0
d n
2. Pd1 Pd 2 = pentru orice d1, d2 divizori naturali distinci ai lui n. Artm
c dac exist Pd1 Pd 2 atunci d1=d2.
ntr-adevr admite scrierile = cos

2k1
2k
2k
2k
+ i sin 2 = cos 2 + i sin 2
d1
d1
d2
d2

unde 0k1d1-1 ; (k1,d1)=1.


2k
2k
2k
2k
2k1 2k 2
Rezult: cos 1 = cos 2 i sin 1 = sin 2 i cum
;
[0,2 )
d1
d2
d1
d2
d1
d2
k
k
deducem 1 = 2 . Deaorece un numr raional pozitiv se reprezint n
d1 d 2
mod unic ca fracie ireductibil cu termeni naturali rezult k1=k2 i d1=d2
z n 1 = ( z ) = ( z ) = (z ) = d ( z ).
U n
d n Pd
d }n
U Pd
d n
Teorema 12. Pentru orice nN*, polinimul n(z) are coeficieni ntregi.
Demonstraie: Pentru n=1 avem 1(z)=z-1Z[z]. Presupunem teorema
z n 1
adevrat pentru toate polinoamele k(z) cu k<n. n ( z ) =
d (z )
d n
d <n

Polinoamele d(z) du d<n sunt, n baza ipotezei de inducie, polinoame cu


coeficieni ntregi, unitare (adic au coeficientul dominant 1). Rezult c
polinomul f = d ( z ) este un polinom unitar cu coeficieni ntregi. Din
d n
d <n

faptul c n(z) este ctul mpririi polinomului zn-1 Z[z] prin polinomul
unitar f rezult n(z)Z[z].

- 49 -

CERCUL DE MATEMATICA
Teorema 13. Fie nN*. Dac m1, m2,,mr numerele naturale mai mici dect
r
r
2m j
2m j
n i prime cu n atunci: sin
= 0 ; cos
Z.
n
n
j =1
j =1
Demonstraie: Polinomul ciclotomic n(z) are rdcinile m1 , m2 ,................., mr .
Deoarece este un polinom unitar cu coeficieni ntregi, din relaiile Viette
m1 , m2 ,................., mr Z .
r

2m j

j =1

cos
r

2m j

j =1

sin
1.
2.
3.
4.
5.
6.

1.
2.
3.
4.
5.
6.
7.

- 50 -

S
S
S
S
S
S

2m j
2m j
r
Z adic cos

n
n
j =1
r
2m j
i cos
Z.
n
j =1

+ i sin

=0

Aplicaii:
se calculeze:
se arate c:
se arate c:
se arate c:
se arate c:
se arate c:

Bibliografie:

2m j
r
+ i sin
j =1
n

Z , de unde

2(z); 3(z); 4(z); 12(z).


z3-1=1(z) 2(z)
z4-1=1(z) 3(z)
z5-1=1(z) 2(z) 3(z) 4(z)
z6-1=1(z) 5(z)
z12-1=1(z) 5(z) 7(z) 11(z)

Dumitru Buneag, Dan Piciu-Lecii de algebr-Ed. Universitaria-Craiova-2002


A. Dinc, Ion D. Ion-Algebr pentru perfecionarea profesorilor-EDP-Bucureti-1983
C. Nstsescu, M. ena-Probleme de structuri algebrice-Ed. Academiei-1988
L.C. Washington-Introduction to cyclotomic fields-Springer-Verlag-New York-1982
V.A. Ilynin, E.G. Poznyak-Linear Algebra-Mir Publishers Moskow-1986
Chahal J. S.-Topics in Number Theory-Plenum Press-1988
Cohen H.-A course in Computational Algebraic Number Theory-Springer Verlag-1995

EXAMENE
BACALAUREAT

SSM
H

Prof. Ciniceanu George

Matematica-informatica
Subiectul I (30p)
Pentru intrebarile 1-10 scrieti litera corespunzatoare raspunsului corect
Fie polinoamele f , g C[X ] , f = x 4 + x 2 + 1 , g = x 2 + 2x + 3
(3p) 1. Restul mpririi lui f la g este:
a) 2x + 5 b) 5x 2
c) 2x 5
d) 5x + 2
e) alt rspuns.
(3p)

(3p)

2.
a)
b)
3.

Funcia polinomial asociat: g : C C a polinomului g este:


injectiv
b) surjectiv
c) monoton
d) bijectiv
e) nici un rspuns.
Descompunerea n factori ireductibili n R [X] a lui f este:

1
3
1
3
1
3
1
3
x +i
x + i
x + +i

a) x i

2
2
2
2
2
2
2
2

b) (x 2 x + 1)(x 2 + x + 1) c) (x 2 + 1)
d) (x 2 x + 1)
e) alt rspuns.
(3p) 4. Dac x1, x2, x3, x4 sunt rdcinile polinomului f atunci suma
x13+x 23+x33+x 43 este:
a) 4
b) 3
c) 0
d) 2
e) 1.
2

(3p)

5. n polinomul f 2004 = (x 4 + x 2 + 1)
coeficientul lui x8010 este:
3
1
2
1
3
0
a) C 2004 C 3 + C 2004 C 2 ;
b) C 2004 C 3 + C 22004 C12 ;
c) C12004 C 32 + C 22004 C13 ;
2004

d) C 32004 C 03 + C 22004 C12 ;

e) alt rspuns.

1
:
( x 1)( x 2)
(3p) 6. Valoarea lui f (0) este egala cu:
a) -0,75
b) 1,25
c) 1,25
d)0,75
e) 1.
3

(3p) 7. Valoarea lui f ( 2003) este egala cu:


2
2003
a) 0
b) (-1)
2003! 22005
c)-2003! 22003
2003
d)2003!
e) (1,5)
(3p) 8. Numrul c obinut prin aplicarea Teoremei lui Lagrange functiei f
pe intervalul [3, 4] satisface egalitatea:
a) 6c-9 = (c-1)2(c 2)2
b) 9c-6=(c-1)2(c-2)2
c)2c+3=(c-1)2(c-2)2
d) 6c-9=(c-1)(c-2) e) 3,5.
(3p) 9. Aria suprafetei plane delimitata de graficul functiei f , dreptele de
ecuatii
y=0 , x=3 , x=4 este egala cu:
3
4
5
a) ln
b) ln
c) ln
d) ln 2
e) 1.
4
3
4

Fie f : R \ {1,2} R , f ( x ) =

n +1

(3p)

10. Valoarea limitei lim n 2


n

a) 2e

b) e -1

f ( x)dx

este:

c) 1

d) 2

e) alt raspuns.

- 51 -

EXAMENE
Pentru subiectele II-IV se cer rezolvarile complete
Subiectul II (20p)
Fie G = {e, a,a1 ..., an-2) un grup cu n elemente in raport cu legea de
compozitie interna notata multiplicativ,, cu proprietatea:
(xy)2 = x2 y2, x , y G . Sa se arate ca
(5p) a) (G , ) este grup comutativ.
(5p) b) Functia f : G G , f(x) = ax este inversabila.
(5p) c) x G , avem ca x n = e .
(5p) d) Fie p N cel mai mic numar natural pentru care a1p = e .Sa se
arate c
p este un divizor al lui n.
Subiectul III (20p)
.
Fie AOBC un tetraedru regulat in reperul ortogonal xOy avand
coordonatele tuturor varfurilor numere nenegative iar O(0,0,0),
C(a,0,0) . Se cer:
(8p) a) Coordonatele punctelor A, B , K (piciorul perpendicularei din A pe
planul
xOz.
(6p) b) Coordonatele vectorilor AK , AB , si unghiul dintre acesti vectori.
(6p) c) Ecuatia planului (ABC) i distanta de la punctul M(0,2a,0) la
planul (ABC).
Subiectul IV (20p)
1
.
2 + sin x
2
a) Sa se calculeze limita lim [f( )] n .
n
n 2
b) Sa se demonstreze ca nu exista limita lim f(x).

Fie functia f : R R , f(x) =


(6p)
(6p)
(8p)

c) Sa se arate ca f admite primitive pe [0,2] si sa se determine

primitiva care se anuleaza in a acestei functii.


2

- 52 -

EXAMENE
Solutii pentru subiectele de la matematica-informatica

SSM
H

I 1) c; 2) b; 3) b; 4) c; 5) b; 6) d; 7) a; 8) a; 9) b; 10) c.
II
a) (xy)2 = x2y 2 deci (xy)(xy)=(xx)(yy). Folosind asociativitatea si
teoremele de simplificare in grup deducem ca yx=xy.
b) Din presupunerea f(x)=f(y) deducem x=y deci functia este injectiva.
Cum domeniul si codomeniul sunt G deci finite , f va fi si surjectiva.
c) Pentru x fixat functia g : GG ,g(t)=xt e bijectie (din b) si deci
g(G)=G. Ca atare si rezultatele operarilor tuturor elementelor:
xexaxa1.xan-2 = eaa1an-2
(am folosit din nou asociativitatea). Deducem prin folosirea esentiala a
comutativitatii si a teoremei de simplificare la dreapta ca
xn = e.
d) Folosind c) avem ca a1n = e = a1p. Daca n=pc+r (0r<p) deducem
succesiv a1pc=e , a1r = e. Deci cu necesitate r=0 si deci p/n.
III
a) Folosind calcule in triunghi echilateral deducem : B(a/2 ,0 ,a 3 /2)
, A(a/2,a 6 /3 ,a 3 /6) , K(a/2 , 0, a 3 /6).
b) AK = a 6 / 3 j , AB = a 6 / 3 j + a 3 / 3k ,cos =

2
.
3

c) Se scrie ecuatia sub forma de determinant


x
y
z 1

xA
xB

yA
yB

zA 1
=0 , si se obtine 6x+ 6 y+2 3 z-6a =0.
zB 1

xC y C z C 1
Folosind formula pentru calculul distantei intre un punct si un plan gasim :
a (6 2 6 )
d(M,(ABC)) =
.
3 6

n 2 (1 cos )
n2
1

n
n

IV
a) lim [f( )] = lim
= lim e
= .. e 2 .
n
n
n

n 2
2 cos
n

1
1

1
1
b) an =n , f(an)= .
bn =(2 n+ ) , f(bn) = .
2
2
2
3
3
c) Functia este continua pe R deci admite primitive pe R si pe [0,2].
x
Se integreaza pe [0,) sau (,2] ,folosind succesiv trecerea la tg si se
2
x
2tg + 1
2
2
arctg
ajunge la
+C. Din conditia de egalitate a limitelor la stanga si
3
3
x
2tg + 1
2

2
la dreapta in deducem ca F(x)=
arctg
+c , pe [0,) ,
+c in ,iar
3
3
3
x
2tg + 1
2
2
2
2
pe intervalul (,2] ,
arctg
+
+c. Se deduce c=.
3
3 3
3
3

- 53 -

EXAMENE
Model de test pentru bacalaureat M2

Subiectul I (30p)
Pentru intrebarile 1-10 scrieti litera corespunzatoare raspunsului corect
Fie f : RR , f(x)=(m-2)x2 +2(2m-3)x + m-2 , unde m este un
parametru real diferit de 2.
(3p) 1. Valorile lui m pentru care ecuatia f(x)=0 are doua solutii reale
distincte sunt :
5
5
3
a) (1 , )
b) (- , 1)( , )\{2}
c) ( , 1)
3
3
5
3
d) (- , )(1 , )\{2}
e) alt raspuns.
5
(3p) 2. Valorile lui m pentru care f are valoare maxima negativa sunt :
5
5
a) (1 , ) b) (1 , ) (2 ,) c) (2 ,)
d) (- ,1)
e) (2 ,15)
3
3
(3p) 3. Daca M={mR | f(x) > 0 , x>0} atunci M este:
a) R\ {2}
b) (2 ,)
c) (-2 ,2)
d) (- ,2)
e)
(3p)

4. Pentru m=3 , multimea solutiilor inecuatiei log 1 ( f(x))< -1 este :


2

a)
(3p)

b) R c) (- ,-3-

10 )(-3+ 10 ) d) (-3- 10 ,-3+ 10 )


e)(1,2)
1
1
1
5. Dac m=3 valoarea determinantului f (0)
f (1)
f (2) este

a) 1124
(3p)
(3p)

(3p)

(3p)

b) 1008

c)1234

[ f (0)] 2
d)1424

[ f (1)] 2 [ f (2)] 2
e) 1524.

Fie sirul (xn) , cu x1=2 si xn+1 = axn +b , unde a,b sunt numere reale.
6. In cazul a=1 , x2004 are valoarea:
a) 2005b
b)2+2004b
c) 2+2003b
d) b+2004
e) b+2003.
7. Pentru b=0 suma primilor n termini ai sirului este:
a n 1
a n 1
a n 1
a)
b) 2
c) 2
d) 2a n-1
e) an
a 1
a +1
a 1
8. Pentru a=2- 3 limita sirului este:
b
b
a)
b)
c) b
d) 0
e) .
3 +1
3 1
9. In care din urmatoarele situatii sirul nu este convergent?
a) a(-1 ,1), bR
b) a>1 , b=2(1-a)
c) a<-1 ,b=2(1-a)
d)a>1,b>0
e) a=0, bR.
2n

(3p)

10. Daca b=1 si a=2 avem ca


1

a) e 3

- 54 -

b) e

1
3

c) e-1

x
lim n +1n este :
n 3 2

d) e

e) 1.

EXAMENE
Pentru subiectele II-IV se cer rezolvarile complete
Subiectul II (20p)
Fie A(2,0),B(5,2),C(0,3),M(2,6) in reperul cartezian xOy.
(5p) a) Se cere ecuatia inaltimii dusa din A a triunghiului ABC.
(7p) b) Sa se determine coordonatele vectorului MB si ecuatia dreptei ce
trece prin A si are vectorul director MB .
(8p) c) Sa se determine raza cercului inscris in triunghiul ABC.

SSM
H

Subiectul III (20p)


a 0 a

Fie ={ Aa= 0 0 0 | aC }.
a 0 a

(5p) a) Sa se arate ca Aa+ Ab=Aa+b ,si AaAb =A2ab.


n

(7p) b) Calculati B= An , n numar natural nenul.


k

k =1

(8p) c) Sa se arate ca (,+,) este inel comutativ fara divizori ai lui 0.


Subiectul IV (20p)
Fie f : RR , f(x)= x 2 + 1 .
(5p) a) Determinati asimptotele la graficul functiei.
(6p) b) Scrieti ecuatia tangentei la graficul functiei in punctul de abscisa 2.
(9p) c)Calculati aria regiunii plane delimitate de grafic , Oy si dreptele de
ecuatii x=1 si x=2.

I
II

Solutii pentru subiectele M2

1) b ; 2) a ; 3) b ; 4) c ; 5) b; 6) c; 7) c; 8) b; 9) d; 10) b.
a) 5x-y-10=0
b) MB =3 i - 4 j . Ecuatia dreptei este 4x+3y-8=0

13(2 2 )
13
2
, semiperimetrul este 13 (1+
) deci r=
2
2
2
a) Prin calcul imediat Aa+Ab=Aa+b , Aa Ab=A2ab.
b)Prin inductie se arata ca Aan = A 2n 1 a n pentu orice n natural nenul. In
c) Aria este

III

1
1
obtinem B = A a ( 2 a 1) 1 ( 2 n a n 1) ,iar pentru a= , B= A n .
2
2
2
c) Din rezolvarea punctului a) deducem ca adunarea si inmultirea
sunt legi interne pe multimea data. Proprietatile de asociativitatea adunarii,
comutativitatea adunarii, asociativitatea inmultirii, distributivitatea
inmultirii fata de adunare sunt valabile in general in M3(C) deci si in acest
caz. Zeroul inelului este Ao , elementul unitate al inelului este A.]
Daca Aa Ab =0 deducem ca 2ab=0 si deci una din matrici este Ao.
IV
a) Are doar asimptote oblice y=-x la - i y=x la .
b) Ecuatia tangentei este 2x-y 5 +1=0.
cazul a

c)

f ( x)dx = 2 (ln(2 +

5 ) + 2 5 ln(1 + 2 ) 2 )

- 55 -

PROBLEME PROPUSE
Clasa a V -a
1. Aflai n tiind c fracia

8 667
2 + 2 + 2 2 + 2 3 + ..... + 2 n

este echiunitar.
prof.Draga Ttucu Porfirel

2. Fie M = 10 n 2 + 1 n N . Artai c M nu conine nici un ptrat perfect.


prof.Constantin Magdalena
3. Aflai valoarea lui x din x + 2 x + 3 x + ..... + 50 x = 505a unde 505a M10 .
prof.Draga Ttucu Mariana
*
3n
2n
3n+1
n
4. Artai c dac nN , atunci: 3 +5 > 2
3
prof.Alexandru Szoros
5. Se dau numerele naturale a = 24m+p i b = 34n+p ; m, n, p N
Aflai p pentru care a+b se divide cu 5.
prof. Iulia i Victor Sceanu
6. Scriei numrul 92004 ca sum de trei ptrate perfecte diferite de zero.
prof. Victor Sceanu
7. Fie numerele a= 3 2 4n+7 5n+1 b =7 24n+4 5 n+2, unde n N
a) Comparai numerele a i b
b) Artai c a i b dau acelai rest la mprirea cu 110.
prof. Victor Sceanu
2
3
8. Artai c numrul A = ( 23n 72 - 23n +2 ) :
este un cub perfect.
5 3 n 1
prof. Victor Sceanu
9. Sa se determine numerele naturale nenule a, b, c, d pentru care
3542001 =a4 + b4 + c4 + d4
prof. Victor Sceanu

Clasa a VI a
1. S se arate c numrul

36 n + 14 6 n + 40
N, (
5 6 n + 50

) nN .
prof. Prenenu Vasile Doru

2. Determinai elementele mulimii A = ab 7 ab = 4 ba .


prof.Draga Ttucu Porfirel
9

3. Fie A = x N
N i B = y N y 2 + y prim . Calculai: A U B i A \ B .
3
x
+
1

prof.Constantin Magdalena
4. Fie ABC cu BC = 20cm , AC = 10cm i M, N astfel nct (BM I ( AN = {C},
CN
2 BM = 3BC iar
= AC .
2
a) Artai c ABC MNC ;
b) Dac AB I MN = P , artai c PBN este isoscel.
prof.Draga Ttucu Mariana

- 56 -

PROBLEME PROPUSE
5. Artai c inversul oricrui numr natural nenul se poate scrie ca suma
inverselor a 7 numere naturale, distincte dou cte dou.
prof.Alexandru Szoros
6. Determinai baza x N pentru ca numrul A = 11...1 ( x ) 22...2 ( x ) s aib un
2 n ori

SSM
H

n ori

numr impar de divizori distinci.

prof. Iulia i Victor Sceanu


7. Cte numere naturale nenule, mai mici ca 10000 nu sunt divizibile cu 1001.
prof. Iulia i Victor Sceanu
8. Fie numerele naturale nenule n1 < n2 < n3 < < nk < n cu (ni ; n) = 1, i=1k si
n
n nk
n
n
n n1 n n2
sirul de fractii 1 ; 2 ; .. k ;
;
;
.
n
n
n
n
n
n
Stabiliti cate fractii sunt ireductibile in sirul dat.
prof. Victor Sceanu
9. Sa se rezolve in multimea numerelor naturale ecuatia 4x +4y +4z =1104,
unde x < y < z.
prof. Victor Sceanu
10. Exista numere naturale de trei cifre divizibile cu 99 care sa aiba toate
cifrele pare ?
prof. Victor Sceanu

Clasa a VII-a
1. S se determine numerele x, y, z , astfel nct x < y < z i
xyz + xy + yz + xz + x + y + z = 1000.
prof. Ovidiu Ticui
n
x
+
2
x
+
...
+
kx
2003
1
2
k
2. Determinai n , a..
=
,
2
1002
k =1 x1 + 2 x 2 + ... + kx k + ... + kx1 + 2kx 2 + ... + k x k
unde x1 , x 2 ,..., x n R + .
3.

4.

5.

6.

prof.Ovidiu Ticui
n( n + 1)
Rezolvai ecuaia ( x 1) k + 1 + ( x 1) k + 4 + ... + ( x 1) k + n 2 =
,
2
x [1, ), k N , n N * .
prof.Ovidiu Ticui
n ( n + 1)

Rezolvai ecuaia [x + 1] + [x + 2] + ... + [x + n ] =


, n , unde [x]
2
reprezint partea intreag a numrului x R .
prof.Ovidiu Ticui
Fie AD nlimea unui triunghi dreptunghic n A. Artai c:
AB AD AC = BD DC BC
prof.Draga Ttucu Porfirel
Latura unui triunghi echilateral este egal cu a. S se calculeze lungimea
segmentului care unete mijloacele a dou nlimi.
prof.Constantin Magdalena

- 57 -

PROBLEME PROPUSE
1
1
1
1

x 1999 x 1972 x 1998 1971


prof.Draga Ttucu Mariana
x y
y z
z x
8. Fie x, y, z numere strict pozitive ce verific: = a ; = b ; = c .
y x
z y
x z
Stabilii o relaie numai ntre a, b i c.
prof.Alexandru Szoros
9. Fie p, qN*. Artai c ecuaia xp+yp+zp = xq+yq+zq +2003 nu are soluii n
mulimea numerelor ntregi.
prof.Alexandru Szoros
10. Fie cercul C(O;R) de diametru AB i M mobil pe cerc, iar P i Q simetricele
lui M fa de A respectiv B. Stabilii poziia lui M astfel nct triunghiul MPQ
s fie isoscel i aflai perimetrul i aria n funcie de R.
prof. Victor Sceanu
11. Fie triunghiul oarecare ABC i M, P (BC) cu MP. Se duc PQ || AB,
Q(AC) i PR || AC, R (AB). Fie {E}=PQ AM. S se arate c AM este
median n triunghiul ABC dac i numai dac PM este median n
triunghiul DEP.
prof. Victor Sceanu
12. n triunghiul ascuitunghic ABC, considerm mediana AM i nlimea
BD. Artai c AM=BD m (< CAM)=300.
prof. Victor Sceanu
13. Fie triunghiul dreptunghic ABC, m(A)=90o,ADBC,D(BC), [BP, P (AD)
bisectoarea unghiului ABC. Demonstrai c paralela prin P la BC trece prin
centrul de greutate G al triunghiului ABC, dac i numai dac m(<C)= 30o.
prof. Victor Sceanu
7. Rezolvai ecuaia:

Clasa a VIII-a
1. Fie x1 , x 2 ,..., x n R + , n . S se arate c
( n! ) 2
, unde n! = 1 2 ... n.
4n
prof.Ovidiu Ticui
n
1
1
2. Fie x1 , x 2 ,..., x n R , n , astfel nct x k
=
.
k (k + 1) n + 1
k =1
x1 x 2 ... x n (1 x1 ) ( 2 x 2 ) ... ( n xn )

S se arate c

x [ 1,1].
k =1

prof.Ovidiu Ticui
3. Fie x, y, z R i expresia E ( x, y, z ) = x 2 + y 2 + z 2 + 2( xy + yz + xz + x + y + z + 1) . S
se determine min E ( x, y, z ) .
x , y , zR

4. Fie x1 , x 2 ,..., x n [k , k + 1], k R, iar n .S se arate c:

prof.Ovidiu Ticui

x12 + x 22 + ... + x n2 ( 2k + 1)( x1 + x 2 + ... + x n ) nk ( k + 1) .

prof.Ovidiu Ticui

- 58 -

PROBLEME PROPUSE
5. Pe planul dreptunghiului ABCD se ridic n vrful D o perpendicular DM.
Dac E este mijlocul lui [MC ] , artai c AM II (EBD ) .
prof.Draga Ttucu Porfirel
6. Determinai mulimea:

SSM
H

8n 2 + 2n + 5

M = n N
N.
2n + 1

prof.Constantin Magdalena

7. Artai c: a = 5 + 3 32 37 1200 N
prof.Draga Ttucu Mariana
8. Fie a, b, cR i expresia E(x)=(x-a)(x-b)+(x-b)(x-c)+(x-c)(x-a).
Artai c E(a)+E(b)+E(c)E(a)E(b)E(c).
prof.Alexandru Szoros
9. Exist (x,y) RxR care verific relaiile: x2+ y2= 7 i x3 + y3= 10 ?
prof. Victor Sceanu
10. In 2n cutii se gasesc x bile colorate in n culori. Stiind ca in fiecare cutie se
gasesc bile de toate culorile, aflati cel mai mic x, astfel incat sa nu avem cutii
cu acelasi numar de bile.
prof. Victor Sceanu

Clasa a-IX-a
1. Artai c: 3 18 + 325 3 18 325 N
prof.Draga Ttucu Porfirel
2. Rezolvai ecuaia:

2 x 3 + 1 3x + x + 2 = 0

3. Rezolvai inecuaia: x 4 + x 2 + 1

prof.Constantin Magdalena

3
x x2 + 1 , x R .
2

prof.Draga Ttucu Mariana


4. Fie a, b, c R cu a 0 . S se arate c dac ecuaia at2 + 2bt + c = 0 are
rdcini reale i distincte, atunci soluiile sistemului:

x 2 + y 2 + z 2 = a
3
3
3
x + y + z = b
4
4
4
x + y + z = c

nu sunt toate reale.

prof.Alexandru Szoros
5. S se arate c o condiie necesar i suficient ca triunghiul ABC s fie
echilateral este ca max{a3,b3,c3 } 4RS.
prof.Alexandru Szoros
6. S se determine valorile lui mR pentru care ecuaia: min ( |2x-1|; |2-x | ) = m,
are patru soluii reale, distincte.
prof. Daniel Sitaru

- 59 -

PROBLEME PROPUSE
[x + y ] = y + z

7. Fie x, y, zR*, astfel nct: [ y + z ] = z + x .


[z + x] = x + y

S se calculeze
8. S

se

E=

(x + y )( y + z )(x + z ) +

demonstreze

xyz

toate

(x + y + z ) 1 + 1 + 1
x
9

rdcinile

prof. Daniel Sitaru


reale
ale
ecuaiei:

x 2n +1 + 2nx n + (2n 1)x 2 n1 x 2 n n 2 = 0; n 1 sunt strict pozitive.

prof. Daniel Sitaru


9. Fie triunghiul isoscel ABC cu AB= AC, M un punct in planul su i
AD MC. Aflai locul geometric al punctelor M care ndeplinesc relaia:
2MD = MB+MC
prof. Iulia i Victor Sceanu
10. Se tie c medianele unui triunghi oarecare ABC, formeaz la rndul lor
un triunghi. Avnd dat triunghiul format de mediane, reconstruii triunghiul
ABC.
prof. Iulia i Victor Sceanu
11. Fie triunghiul oarecare ABC, (AM, AN trisectoarele unghiului A, iar R1,
R2, R3 si R razele cercurilor circumscrise triunghiurilor BAM, MAN, NAC si
respectiv ABC. Aratati ca triunghiul ABC este dreptunghic in A daca si numai
daca: R1+ R2+ R3 =2R
prof. Iulia i Victor Sceanu
12. In triunghiul ABC, fie inaltimile AA, BB, CC, care indeplinesc conditia
1
1
1
+
+
=1. Aflati raza cercului inscris in triunghiul ABC.
'
'
AA BB CC '
prof. Iulia i Victor Sceanu
13. Daca a, b, c R, sa se calculeze minimul expresiei
17
17
17
E = a 2 + b 2 + c 2 ab bc ac + 1
4
4
4
prof. Iulia i Victor Sceanu
14. Sa se arate ca in orice triunghi ascutitunghic are loc inegalitatea:
b
c 1
1
1
1
a
1
1
2
+
+
+
+
+

c +
b +
a
cos A cos B cos C cos A cos B
cos A cos C
cos C cos B
prof. Nedeianu Dan

Clasa a-X-a
1. Artai c ecuaia: z 3 + (2i 5)z 2 + 7(1 i )z 2 + 6i = 0 are o rdcin real.
Aflai celelalte rdcini.
prof.Draga Ttucu Porfirel

- 60 -

PROBLEME PROPUSE

2. Rezolvai ecuaia: x3 m 2 m + 1 x + m m 2 = 0 , m R .
prof.Constantin Magdalena
3. S se arate c numrul

SSM
H

(n )! este ntreg.
2

(n!)n

prof.Draga Ttucu Mariana


B
b+a C c+a
4. Precizai natura ABC n care are loc relaia:
sin +
sin = 1 .
2 c+b
2
b+c
prof.Alexandru Szoros
5. S se arate c n orice triunghi A1A2A3 avem:
2
A
A
A
4 cos 1 cos 2 cos 3 33 sin A1 sin A2 sin A3 max sin Ai sin A j
1

i
<
j

3
2
2
2

prof. Daniel Sitaru


6. S se arate c dac zC, |z| <1/2 atunci: (1 + i )z 3 iz 2 + 2 2 i < 4
prof. Daniel Sitaru
7. S se arate c dac ntr-un triunghi razele cercurilor exnscrise sunt
proporionale cu laturile corespunztoare, triunghiul este echilateral.
Rmne valabil rspunsul pentru nlimi? Dar pentru mediane? Dar pentru
bisectoarele interioare?
prof. Daniel Sitaru
8. S se reprezinte n planul P mulimea:

z +1
z +1
Im
; z = x + iy
( x, y ) P Re
z + 2
z + 2

prof. Daniel Sitaru


a
b
b
a
9. Daca a , b R si log5(2 +3)<log2(5 -3) si log8(5 +3)>log5(8 -3), sa se compare
numerele a si b.
prof. Nedeianu Dan

Clasa a-XI-a
1. Fie ( xn ) n 0 R + un ir de numere astfel nct: e xn = sin( x n + 1), n 0 .
S se arate c lim x n = 0 .
n

prof.Ovidiu Ticui
2. Fie a R i funciile f i , g i : [ a, a ] R , i = 1, n , n , cu proprietile:

i) f i , g i derivabile pe [ a, a ], i = 1, n

ii) g i sunt funcii pare,


S se arate c:

i = 1, n

i =1

i 1

iii ) f i ( x) g i ( x) f i (0) g i (0).

f (0) g (0) = 0.
i =1

prof.Ovidiu Ticui

- 61 -

PROBLEME PROPUSE
1 2 3

3. Fie A = 0 1 3 . Calculai An , n N * .
0 0
1

prof.Draga Ttucu Porfirel


n

k k !

4. Calculai: lim

k =1

2(n + 1) !
prof.Constantin Magdalena
n

sin 2 k

5. Calculai: lim

k =1

2n

prof.Draga Ttucu Mariana

6. Fie AMn( C ). S se arate c:

(det A )A + 3 det A(A )


*

* *

+ 6(det A)

n 2

A 2 A* = 10(det A)

n 1

A
prof. Daniel Sitaru

7. S se ordoneze cresctor numerele:


1
1

e ; e i e unde = lim 1 + + .......... + ln n .


n
n
2

prof. Daniel Sitaru


8. Fie X o variabil aleatoare avnd distribuia:
1 2
3 . . . . . n
0
1
1 ; n 1.
X ( ) 1 1
.
.
.
.
.
p n

3 3 2 33
3n

S se calculeze lim p(n ) i lim M ( x ) .


n

prof. Daniel Sitaru

1
1
1

9. S se calculeze: lim sh
+ sh
+ ........... + sh .
n
n+2
2n
n +1
prof. Daniel Sitaru
10.

Fie A, B M n (R) astfel incat AB=On. Demonstrati ca

det(A2+B2+A+B+In)0.

prof. Nedeianu Dan

Clasa a-XII-a
1. Calculai:

sin 3x cos 5 xdx

2. Calculai: cos 2 x cos 4 x cos 5xdx

prof.Draga Ttucu Porfirel


prof.Constantin Magdalena

3. a) S se determine a, b, c Z 5 a. . X 4 + aX 3 + bX 2 + 2 X + 4 = X 2 + cX + 2 ;

- 62 -

PROBLEME PROPUSE
b) Rezolvai ecuaia: X 2 + cX + 2 = 0 unde c este aflat din a).
prof.Draga Ttucu Mariana
1

SSM
H

4. Pentru x[0,1] demonstrai c: 1 + x 2 e t dt + 2t 2 e t dt + (2t 2 2t )e t dt e .


2

prof.Alexandru Szoros
5. S se arate c dac f:[a,b][a,b] este o funcie bijectiv i monoton atunci:
b

( f o f )(x )dx + ( f

o f 1 ( y )dy = b 2 a 2 .

prof. Daniel Sitaru


x
6. S se determine matricile A =
0
4
3
2
A +3A +2A +3A+I2=O2 .
kN*;

7. Fie a, bR ;

y
; xR*; yR, care verific relaia:
x

prof. Daniel Sitaru


k2; k impar. Definim pe R legile de compoziie:

x * y = k x + k y k a i x o y = k x + k y k b . S se arate c (R,*) i (R,) sunt


grupuri abeliene izomorfe i s se gseasc izomorfismul corespunztor.
prof. Daniel Sitaru
x

8. Se consider polinoamele

(Pn)nN*

date de P1 ( x ) = 1; Pn +1 ( x ) = Pn ( y )dy. S se
2

arate c polinomul Tn (x ) =
1

9. Calculati: I =

P (x ) nu poate avea rdcini multiple.


k =0

k +1

( x + 1) ln( x + 1)
dx.
2
x + 4x + 5 x 2 + 1

)(

prof. Daniel Sitaru

prof. Nedeianu Dan

- 63 -

REZOLVITORI

COALA NR.11
Prof. Victor Sceanu
clasa a V-a Avramescu Alexandra (10), Bcioiu Loredana (10), Bdi
Bogdan (10), Cimpoeru Diana (10), Moneanu Angela (10), Poenaru Ctlin
(10), Roman Georgiana (10), Truic Cristian (10), Zaharescu Eduard (10).
clasa a VI-a Avram Andreea (4), Bratu Simona (4), Ciupagea Florin (4),
Dragomir Adelina (4), Betelei Claudia (4), Petrescu Simona (4), Rogobete
Bogdan (4), Stoican Elena (4), Vasilescu Ovidiu (4), Vi;an Cosmina (4).

CNT
Prof. Paponiu Dana
clasa a-V-a Pupaza Elena (11), Pruna Cristina (5), Barbulescu Andrada (13),
Paraschiv Arina (5), Ecobici Gabriela (12), Diaconescu Alina (11), Cioarec
Roxana (7).
clasa a VIII-a Pirvulescu Cristina (6), Olteanu Liliana (7), Soare Monica (8),
Rosu Stefan (8), Plotogea Ilie (8), Meilescu Simona (8), Bejinaru Amalia (8).
Prof. Gh. Cainiceanu
clasa a-VIII-a Tuta Leontin (5), Negrea Nicolae (5), Stefan Luminita (6),
Ceraceanu Bianca (6).
Clasa a-XI-a Badescu Adrian (5), Mandroiu Sebastian (6), Cocosila Radu (6).
Clasa a-XII-a Cioana Georgiana (7), Nechita Alina (7), Gherasim Bogdan (7),
Pop Andrei (7), Murgea Dan (7), Popescu Iulia (6), Vilceanu Beatrice (6),
Braila Iulia (7), Daescu Catalina (7).

- 64 -

COLABORATORI

1
2
3
4
5
6
7
8
9
10
11
12
13
14
15
16
17

SSM
H

Adela Ionescu
Alexandru Szoros
Ani Pavelescu
Constantin Duu
Dan Nedeianu
Daniel Sitaru
Doru Preneanu
Florentina Gabriela Roman
Gheorghe Cainiceanu
Iulia Sceanu
Magdalena Constantin
Manuela Prajea
Mariana Draga Ttucu
Ovidiu Ticui
Pi Rada Ionel Vasile
Porfirel Draga Ttucu
Victor Sceanu

Colegiul Universitar
coala general nr. 2
Grup colar Dl. Tudor
Colegiul Naional Economic
Colegiul Naional Traian
Colegiul Naional Traian
Liceul Gh. ieica
Colegiul Naional Traian
coala general nr. 11

Colectivul de redacie:
Constantin Ptrcoiu
Octavian Ungureanu
Eleodor Popescu

Gheorghe Ciniceanu
Manuela Prajea
Dan Daniel

Decembrie 2003

- 65 -

You might also like